Prep U-Chap 51-Assessment and Management of Patients with Diabetes

Réussis tes devoirs et examens dès maintenant avec Quizwiz!

Which clinical manifestation of type 2 diabetes occurs if glucose levels are very high? a. Hyperactivity b. Blurred vision c. Oliguria d. Increased energy

Blurred vision Blurred vision occurs when blood glucose levels are very high. The other clinical manifestations are not consistent with type 2 diabetes.

Which of the following are byproducts of fat breakdown, which accumulate in the blood and urine?

Ketones Explanation: Ketones are byproducts of fat breakdown, and they accumulate in the blood and urine. Creatinine, hemoglobin, and cholesterol are not byproducts of fat breakdown.

An 18-year-old female client, 5′4′′ tall, weighing 113 kg, comes to the clinic for a nonhealing wound on her lower leg, which she has had for two (2) weeks. Which disease process should the nurse suspect the client has developed? 1. Type 1 diabetes. 2. Type 2 diabetes. 3. Gestational diabetes. 4. Acanthosis nigricans.

2. Type 2 diabetes.

Which assessment finding is most important in determining nursing care for a client with diabetes mellitus?

Fruity breath

A client with diabetes mellitus is receiving an oral antidiabetic agent. The nurse observes for which symptom when caring for this client? a. Polyuria b. Hypoglycemia c. Blurred vision d. Polydipsia

Hypoglycemia The nurse should observe the client receiving an oral antidiabetic agent for signs of hypoglycemia. The time when the reaction might occur is not predictable and could be from 30 to 60 minutes to several hours after the drug is ingested. Polyuria, polydipsia, and blurred vision are symptoms of diabetes mellitus.

The nurse is explaining glycosylated hemoglobin testing to a diabetic client. Which of the following provides the best reason for this order? a) Reflects the amount of glucose stored in hemoglobin over past several months. b) Provides best information on the body's ability to maintain normal blood functioning c) Is less costly than performing daily blood sugar test d) Best indicator for the nutritional state of the client

a) Reflects the amount of glucose stored in hemoglobin over past several months.

Which is the best nursing explanation for the symptom of polyuria in a client with diabetes mellitus?

High sugar pulls fluid into the bloodstream, which results in more urine production.

A nurse in the surgical ICU just received a client from recovery following a Whipple procedure. Which nursing diagnoses should the nurse consider when caring for this acutely ill client? Select all that apply.

potential for infection acute pain and discomfort alterations in respiratory function

Which are consistent as causes of DKA? A. Illness or infection B. Competency in injecting insulin C. Undiagnosed and untreated diabetes D. Decreased or missed dose of insulin

undiagnosed and untreated diabetes Being able to competently inject insulin is not a cause of DKA. Undiagnosed and untreated diabetes decreased or missed dose of insulin, and illness or infection are potential causes of DKA.

When administering insulin to a client with type 1 diabetes, which of the following would be most important for the nurse to keep in mind? A. Technique for injecting B. Accuracy of the dosage C. Duration of the insulin D. Area for insulin injection

Accuracy of the dosage The measurement of insulin is most important and must be accurate because clients may be sensitive to minute dose changes. The duration, area, and technique for injecting should also to be noted.

Which of the following medications is considered a glitazone?

Actos Explanation: Pioglitazone (Actos) and rosiglitazone (Avandia) are classified as a glitazone or thiazolidinedione. Metformin (Glucophage, Glucophase) and metformin with glyburide (Glucovance) are classified as biguanides.

A nurse is caring for a diabetic patient with a diagnosis of nephropathy. What would the nurse expect the urinalysis report to indicate? A. White blood cells B. Red blood cells C. Bacteria D. Albumin

Albumin Albumin is one of the most important blood proteins that leak into the urine. Although small amounts may leak undetected for years, its leakage into the urine is among the earliest signs that can be detected. Clinical nephropathy eventually develops in more than 85% of people with microalbuminuria but in fewer than 5% of people without microalbuminuria. The urine should be checked annually for the presence of microalbumin.

A client has received a diagnosis of type 2 diabetes. The diabetes nurse has made contact with the client and will implement a program of health education. What is the nurse's priority action?

Assess the client's readiness to learn.

Which of the following clinical characteristics is associated with type 2 diabetes (previously referred to as non-insulin dependent diabetes mellitus [NIDDM])?

Can control blood glucose through diet and exercise Explanation: Oral hypoglycemic agents may improve blood glucose levels if dietary modification and exercise are unsuccessful. Individuals with type 2 diabetes are usually obese at diagnosis. Individuals with type 2 diabetes rarely demonstrate ketosis, except with stress or infection. Individuals with type 2 diabetes do not demonstrate islet cell antibodies.

A client with a serum glucose level of 618 mg/dl is admitted to the facility. He's awake and oriented, has hot dry skin, and has the following vital signs: temperature of 100.6° F (38.1° C), heart rate of 116 beats/minute, and blood pressure of 108/70 mm Hg. Based on these assessment findings, which nursing diagnosis takes highest priority?

Deficient fluid volume related to osmotic diuresis

Which information should be included in the teaching plan for a client receiving glargine, which is "peakless" basal insulin?

Do not mix with other insulins. Because glargine is in a suspension with a pH of 4, it cannot be mixed with other insulins because this would cause precipitation. Glargine is a "peakless" basal insulin that is absorbed very slowly over a 24-hour period and can be given once a day.

A nurse is providing health education to an adolescent newly diagnosed with type 1 diabetes mellitus and her family. The nurse teaches the client and family the twhat nonpharmacologic measures will decrease the body need for insulin?

Exercise

A client has an elevated serum ammonia concentration and is exhibiting changes in mental status. The nurse should suspect which condition?

Hepatic encephalopathy

Why is a by-product of fat breakdown in the absence of insulin and accumulates in the blood and urine?

Ketones

The nurse is preparing to administer intermediate-acting insulin to a patient with diabetes. Which insulin will the nurse administer? a) Lispro (Humalog) b) Glargine (Lantus) c) NPH d) Iletin II

NPH Intermediate-acting insulins are called NPH insulin (neutral protamine Hagedorn) or Lente insulin. Lispro (Humalog) is rapid acting, Iletin II is short acting, and glargine (Lantus) is very long acting.

A client who was diagnosed with type 1 diabetes 14 years ago is admitted to the medical-surgical unit with abdominal pain. On admission, the client's blood glucose level is 470 mg/dl. Which finding is most likely to accompany this blood glucose level? A. Arm and leg trembling B. Rapid, thready pulse C. Slow, shallow respirations D. Cool, moist skin

Rapid, thready pulse This client's abnormally high blood glucose level indicates hyperglycemia, which typically causes polyuria, polyphagia, and polydipsia. Because polyuria leads to fluid loss, the nurse should expect to assess signs of deficient fluid volume, such as a rapid, thready pulse; decreased blood pressure; and rapid respirations. Cool, moist skin and arm and leg trembling are associated with hypoglycemia. Rapid respirations — not slow, shallow ones — are associated with hyperglycemia.

What is the only insulin that can be given intravenously? A. Regular B. Ultralente C. Lantus D. NPH

Regular Insulins other than regular are in suspensions that could be harmful if administered IV.

A client is admitted to the unit with diabetic ketoacidosis (DKA). Which insulin would the nurse expect to administer intravenously? a) Glargine b) Lente c) Regular d) NPH

Regular Regular insulin is administered intravenously to treat DKA. It is added to an IV solution and infused continuously. Glargine, NPH, and Lente are only administered subcutaneously.

A client with type 2 diabetes has been managing his blood glucose levels using diet and metformin. Following an ordered increase in the client's daily dose of metformin, the nurse should prioritize which of the following assessments?

Reviewing the client's creatinine and BUN levels

A patients been newly diagnosed with type 2 diabetes, and the nurse is assisting with the development of a meal plan. What step should be taken into consideration prior to making the meal plan?

Reviewing the patients diet history to identify eating habits and lifestyle and cultural eating patterns

The nurse expects that a type 1 diabetic patient may receive what percentage of his or her usual morning dose of insulin preoperatively? a. 10% to 20% b. 25% to 40% c. 50% to 60% d. 85% to 90%

c. 50% to 60%

A diabetic nurse is working for the summer at a camp for adolescents with diabetes. When providing information on the prevention and management of hypoglycemia, what action should the nurse promote? a) Eat a meal or snack every 8 hours. b) Check blood sugar at least every 24 hours. c) Always carry a form of fast-acting sugar. d) Perform exercise prior to eating whenever possible.

c) Always carry a form of fast-acting sugar.

A client with status asthmaticus requires endotracheal intubation and mechanical ventilation. Twenty-four hours after intubation, the client is started on the insulin infusion protocol. The nurse must monitor the client's blood glucose levels hourly and watch for which early signs and symptoms associated with hypoglycemia? a) Sweating, tremors, and tachycardia b) Dry skin, bradycardia, and somnolence c) Polyuria, polydipsia, and polyphagia d) Bradycardia, thirst, and anxiety

Sweating, tremors, and tachycardia

A client with type 1 diabetes asks the nurse about taking an oral antidiabetic agent. The nurse explains that these medications are effective only if the client:

has type 2 diabetes.

A client has received a diagnosis of portal hypertension. What does portal hypertension treatment aim to reduce? Select all that apply.

fluid accumulation venous pressure

A nurse educator been invited to local seniors center to discuss health-maintaining strategies for older adults. The nurse addresses the subject of diabetes mellitus, its symptoms, and consequences. What should the educator teach the participants about type 1 diabetes?

The participants are unlikely to develop a new onset of type 1 diabetes. Type 1 diabetes usually (but not always) develops in people younger than 20. In older adults, an onset of type 2 is far more common. A significant number of older adults develops type 2 diabetes.

Which intervention is essential when performing dressing changes on a client with a diabetic foot ulcer? A. Applying a heating pad B. Debriding the wound three times per day C. Using sterile technique during the dressing change D. Cleaning the wound with a povidone-iodine solution

Using sterile technique during the dressing change The nurse should perform the dressing changes using sterile technique to prevent infection. Applying heat should be avoided in a client with diabetes mellitus because of the risk of injury. Cleaning the wound with povidone-iodine solution and debriding the wound with each dressing change prevents the development of granulation tissue, which is essential in the wound healing process.

A patient who is 6 months' pregnant was evaluated for gestational diabetes mellitus. The doctor considered prescribing insulin based on the serum glucose result of: 90 mg/dL before meals. 138 mg/dL, 2 hours postprandial. 80 mg/dL, 1 hour postprandial. 120 mg/dL, 1 hour postprandial.

138 mg/dL, 2 hours postprandial. The goals for a 2-hour, postprandial blood glucose level are less than 120 mg/dL in a patient who might develop gestational diabetes.

A nurse knows to assess a patient with type 1 diabetes for postprandial hyperglycemia. The nurse knows that glycosuria is present when the serum glucose exceeds:

180 mg/dl

Insulin is secreted by which of the following types of cells?

Beta cells

A nurse is providing dietary instructions to a client with hypoglycemia. To control hypoglycemic episodes, the nurse should recommend: a) increasing intake of vitamins B and D and taking iron supplements. b) consuming a low-carbohydrate, high-protein diet and avoiding fasting. c) increasing saturated fat intake and fasting in the afternoon. d) eating a candy bar if light-headedness occurs.

b) consuming a low-carbohydrate, high-protein diet and avoiding fasting. To control hypoglycemic episodes, the nurse should instruct the client to consume a low-carbohydrate, high-protein diet, avoid fasting, and avoid simple sugars. Increasing saturated fat intake and increasing vitamin supplementation wouldn't help control hypoglycemia.

The charge nurse is making client assignments in the intensive care unit. Which client should be assigned to the most experienced nurse? 1. The client with type 2 diabetes who has a blood glucose level of 348 mg/dL. 2. The client diagnosed with type 1 diabetes who is experiencing hypoglycemia. 3. The client with DKA who has multifocal premature ventricular contractions. 4. The client with HHNS who has a plasma osmolarity of 290 mOsm/L.

3. The client with DKA who has multifocal premature ventricular contractions.

The emergency department nurse is caring for a client diagnosed with HHNS who has a blood glucose of 680 mg/dL. Which question should the nurse ask the client to determine the cause of this acute complication? 1. "When is the last time you took your insulin?" 2. "When did you have your last meal?" 3. "Have you had some type of infection lately?" 4. "How long have you had diabetes?"

3. "Have you had some type of infection lately?"

The client diagnosed with type 2 diabetes is admitted to the intensive care unit with hyperosmolar hyperglycemic nonketonic syndrome (HHNS) coma. Which assessment data should the nurse expect the client to exhibit? 1. Kussmaul's respirations. 2. Diarrhea and epigastric pain. 3. Dry mucous membranes. 4. Ketone breath odor.

3. Dry mucous membranes.

Which of the following would be included in the teaching plan for a patient diagnosed with diabetes mellitus? a) Sugar is found only in dessert foods. b) Once insulin injections are started in the treatment of type 2 diabetes, they can never be discontinued. c) Elevated blood glucose levels contribute to complications of diabetes, such as diminished vision. d) The only diet change needed in the treatment of diabetes is to stop eating sugar.

c) Elevated blood glucose levels contribute to complications of diabetes, such as diminished vision. Explanation: Diabetic retinopathy is the leading cause of blindness among people between 20 and 74 years of age in the United States; it occurs in both type 1 and type 2 diabetes. When blood glucose levels are well controlled, the potential for complications of diabetes is reduced.

A patient receives a daily injection of glargine (Lantus) insulin at 7:00 am. When should the nurse monitor this patient for a hypoglycemic reaction? a) Between 7:00 and 9:00 pm b) Between 4:00 and 6:00 pm c) Between 8:00 and 10:00 am d) This insulin has no peak action and does not cause a hypoglycemic reaction.

d) This insulin has no peak action and does not cause a hypoglycemic reaction. Explanation: Peakless basal or very long-acting insulins are approved by the Food and Drug Administration for use as a basal insulin—that is, the insulin is absorbed very slowly over 24 hours and can be given once a day. It has is no peak action.

A patient with type 1 diabetes mellitus is being taught about self-injection of insulin. Which of the following facts about site rotation should the nurse include in the teaching? a) Avoid the abdomen because absorption there is irregular. b) Rotate sites from area to area every other day. c) Choose a different site at random for each injection. d) Use all available injection sites within one area.

d) Use all available injection sites within one area. Explanation: Systematic rotation of injection sites within an anatomic area is recommended to prevent localized changes in fatty tissue. To promote consistency in insulin absorption, the patient should be encouraged to use all available injection sites within one area rather than randomly rotating sites from area to area.

A nurse is developing a teaching plan for a client with diabetes mellitus. A client with diabetes mellitus should: a) use commercial preparations to remove corns. b) walk barefoot at least once each day. c) cut the toenails by rounding edges. d) wash and inspect the feet daily.

d) wash and inspect the feet daily. A client with diabetes mellitus should wash and inspect his feet daily and should wear nonconstrictive shoes. Corns should be treated by a podiatrist — not with commercial preparations. Nails should be filed straight across. Clients with diabetes mellitus should never walk barefoot.

As a nurse educator, you have been invited to your local senior center to discuss health-maintaining strategies for older adults. During your education session on nutrition, you approach the subject of diabetes mellitus, its symptoms and consequences. What is the prevalence of Type I diabetes?

5% to 10% of all diagnosed cases Explanation: Type 1 diabetes accounts for approximately 5% to 10% of all diagnosed cases of diabetes (National Institute of Diabetes and Digestive and Kidney Diseases [NIDDK], 2008).

A diabetes nurse educator is presenting current recommendations for levels of caloric intake. What should the nurse describe?

50% to 60% fo calories from carbohydrates, 20% to 30% from fat, and there remaining 10% to 20% from protein

A 16-year-old client newly diagnosed with type 1 diabetes has a very low body weight despite eating regular meals. The client is upset because friends frequently state, "You look anorexic." Which statement by the nurse would be the best response to help this client understand the cause of weight loss due to this condition? 1 "Don't worry about what your friends think; the carbohydrates you eat are being quickly digested, increasing your metabolism." 2 "Your body is using protein and fat for energy instead of glucose." 3 "I will refer you to a dietician who can help you with your weight." 4 "You may be having undiagnosed infections, causing you to lose extra weight."

delete

The nurse is caring for a client with an abnormally low blood glucose concentration. What glucose level will the nurse observe when assessing laboratory results?

Lower than 50 to 60 mg/dL (2.77 to 3.33 mmol/L)

A client is taking glyburide (DiaBeta), 1.25 mg P.O. daily, to treat type 2 diabetes. Which statement indicates the need for further client teaching about managing this disease? a) "I skip lunch when I don't feel hungry." b) "I avoid exposure to the sun as much as possible." c) "I always wear my medical identification bracelet." d) "I always carry hard candy to eat in case my blood sugar level drops."

"I skip lunch when I don't feel hungry."

A newly admitted client with type 1 diabetes asks the nurse what caused her diabetes. When the nurse is explain to the client the etiology of type diabetes, what process should the nurse describe?

"destruction of special cells in the pancreas causes a decrease in insulin production. Glucose levels rise because insulin normally break it down."

The client diagnosed with type 1 diabetes is found lying unconscious on the floor of the bathroom. Which intervention should the nurse implement first? 1. Administer 50% dextrose IVP. 2. Notify the health-care provider. 3. Move the client to the ICU. 4. Check the serum glucose level.

1. Administer 50% dextrose IVP.

What is the duration of regular insulin? A. 12 to 16 hours B. 24 hours C. 4 to 6 hours D. 3 to 5 hours

4 to 6 hours The duration of regular insulin is 4 to 6 hours; 3 to 5 hours is the duration for rapid-acting insulin such as Novolog. The duration of NPH insulin is 12 to 16 hours. The duration of Lantus insulin is 24 hours.

Which clinic characteristic is associated with type 2 diabetes (previously referred to as non-insulin-dependent diabetes mellitus)?

Blood glucose can be controlled though diet and exercise

A client with type 1 diabetes reports waking up in the middle of the night feeling nervous and confused, with tremors, sweating, and a feeling of hunger. Morning fasting blood glucose readings have been 110 to 140 mg/dL. The client admits to exercising excessively and skipping meals over the past several weeks. Based on these symptoms, the nurse plans to instruct the client to

Check blood glucose at 3:00 a.m. In the Somogyi effect, the client has a normal or elevated blood glucose concentration at bedtime, which decreases to hypoglycemic levels at 2 to 3 a.m., and subsequently increases as a result of the production of counter-regulatory hormones. It is important to check blood glucose in the early morning hours to detect the initial hypoglycemia.

Several clinical manifestations are associated with a tumor of the head of the pancreas. Choose all that apply.

Clay-colored stools Dark urine Jaundice

A client's blood glucose level is 45 mg/dl. The nurse should be alert for which signs and symptoms? a) Polyuria, polydipsia, hypotension, and hypernatremia b) Coma, anxiety, confusion, headache, and cool, moist skin c) Polyuria, polydipsia, polyphagia, and weight loss d) Kussmaul's respirations, dry skin, hypotension, and bradycardia

Coma, anxiety, confusion, headache, and cool, moist skin Signs and symptoms of hypoglycemia (indicated by a blood glucose level of 45 mf/dl) include anxiety, restlessness, headache, irritability, confusion, diaphoresis, cool skin, tremors, coma, and seizures. Kussmaul's respirations, dry skin, hypotension, and bradycardia are signs of diabetic ketoacidosis. Excessive thirst, hunger, hypotension, and hypernatremia are symptoms of diabetes insipidus. Polyuria, polydipsia, polyphagia, and weight loss are classic signs and symptoms of diabetes mellitus.

Which of the following is a characteristic of diabetic ketoacidosis (DKA)? Select all that apply.

Elevated blood urea nitrogen (BUN) and creatinine Rapid onset More common in type 1 diabetes

Which is the primary dietary consideration for a client receiving insulin isophane suspension (NPH) at breakfast?

Encourage midday snack.

Which of the following insulins are used for basal dosage? A. Lispro (Humalog) B. Aspart (Novolog) C. NPH (Humulin N) D. Glarginet (Lantus)

Glarginet (Lantus) Lantus is used for basal dosage. NPH is an intermediate acting insulin, usually taken after food. Humalog and Novolog are rapid-acting insulins.

A student with diabetes tells the school nurse that he is feeling nervous and hungry. The nurse assesses the child and finds he has tachycardia and is diaphoretic with a blood glucose level of 50 mg/dL (2.8 mmol/L). What should the school nurse administer?

Half of a cup of juice, followed by cheese and crackers

Which of the following is the most rapid acting insulin?

Humalog Explanation: The onset of action of rapid-acting Humalog is within 10 to 15 minutes. The onset of action of short-acting regular insulin is 30 minutes to 1 hour. The onset of action of intermediate acting NPH is 3 to 4 hours. The onset of action of long-acting Ultralente is 6 to 8 hours.

Exercise lowers blood glucose levels. Which of the following are the physiologic reasons that explains this statement. Select all that apply.

Increase lean muscle mass, increase resting metabolic rate as muscle size increases, Decreases total cholesterol, increases glucose uptake by body muscles

A client with type 1 diabetes is admitted to an acute care facility with diabetic ketoacidosis. To correct this acute diabetic emergency, which measure should the health care team take first?

Initiate fluid replacement therapy.

The nurse is preparing a presentation for a group of adults at a local community center about diabetes. Which of the following would the nurse include as associated with type 2 diabetes? A. Insufficient insulin production B. Little relation to prediabetes C. Less common than type 1 diabetes D. Onset most common during adolescence

Insufficient insulin production Type 2 diabetes is characterized by insulin resistance or insufficient insulin production. It is more common in aging adults and now accounts for 20% of all newly diagnosed cases. Type 1 diabetes is more likely in childhood and adolescence; although, it can occur at any age. It accounts for approximately 5% to 10% of all diagnosed cases of diabetes. Prediabetes can lead to type 2 diabetes.

Which statement is correct regarding glargine insulin? a) Its peak action occurs in 2 to 3 hours. b) It is absorbed rapidly. c) It cannot be mixed with any other type of insulin. d) It is given twice daily.

It cannot be mixed with any other type of insulin.

A client is scheduled to have a laparoscopic cholecystectomy as an outpatient. The client asks the nurse when he will be able to resume normal activities. What information should the nurse provide?

Normal activities may be resumed in 1 week.

Lispro (Humalog) is an example of which type of insulin?

Rapid-acting

Lispro (Humalog) is an example of which type of insulin? a) Short-acting b) Intermediate-acting c) Rapid-acting d) Long-acting

Rapid-acting Humalog is a rapid-acting insulin. NPH is an intermediate-acting insulin. A short-acting insulin is Humulin-R. An example of a long-acting insulin is Glargine (Lantus).

The nurse must monitor the clients blood glucose levels hourly and watch for which early signs and sx associated with hypoglycemia?

Sweating, tremors, and tachycardia

A nurse educator has been invited to speak with a group of parents about type 2 diabetes in children. The nurse should emphasize:

The cold of obesity in causing diabetes

When the nurse is caring for a patient with type 1 diabetes, what clinical manifestation would be a priority to closely monitor? a. hypoglycemia b. hyponatremia c. ketonuria d. polyphagia

A. hypoglycemia

A client who is suspected of having diabetes is undergoing a postprandial glucose test. Which result would the nurse interpret as suggestive of diabetes? a) 70 mg /dL b) 110 mg/dL c) 220 mg/dL d) 160 mg/dL

160 mg/dL

The nurse is providing information about foot care to a client with diabetes. Which of the following would the nurse include? A. "Be sure to apply a moisturizer to feet daily." B. "Wash your feet in hot water every day." C. "Use a razor to remove corns or calluses." D. "Wear well-fitting comfortable rubber shoes."

"Be sure to apply a moisturizer to feet daily." The nurse should advise the client to apply a moisturizer to the feet daily. The client should use warm water not hot water to bathe his feet. Razors to remove corns or calluses must be avoided to prevent injury and infection. The client should wear well-fitting comfortable shoes, avoiding shoes made of rubber, plastic or vinyl which would cause the feet to perspire.

A client newly diagnosed with type 1 diabetes has an unusual increase in blood glucose from bedtime to morning. The physician suspects the client is experiencing insulin waning. Based on this diagnosis, the nurse expects which change to the client's medication regimen?

Administering a dose of intermediate-acting insulin before the evening meal

A patient is suspected to have pancreatic carcinoma and is having diagnostic testing to determine insulin deficiency. What would the nurse determine is an indicator for insulin deficiency in this patient? (Select all that apply).

An abnormal glucose tolerance Glucosuria Hyperglycemia

A client with cirrhosis has a massive hemorrhage from esophageal varices. Balloon tamponade is used temporarily to control hemorrhage and stabilize the client. In planning care, the nurse gives the highest priority to which goal?

Maintaining the airway

A patient who is diagnosed with type 1 diabetes would be expected to: A. Be restricted to an American Diabetic Association diet. B. Have no damage to the islet cells of the pancreas. C. Receive daily doses of a hypoglycemic agent. D. Need exogenous insulin.

Need exogenous insulin Type 1 diabetes is characterized by the destruction of pancreatic beta cells that require exogenous insulin.

A nurse is preparing to administer two types of insulin to a client with diabetes mellitus. What is the correct procedure for preparing this medication?

The short-acting insulin is withdrawn before the intermediate-acting insulin. When combining two types of insulin in the same syringe, the short-acting regular insulin is withdrawn into the syringe first and the intermediate-acting insulin is added next. This practice is referred to as "clear to cloudy."

Which of the following is an age-related change that may affect diabetes and its management? a) Increased bowel motility b) Decreased renal function c) Increased thirst d) Hypotension

b) Decreased renal function Explanation: Decreased renal function affects the management of diabetes. With decreasing renal function, it takes longer for oral hypoglycemic agents to be excreted by the kidneys and changes in insulin clearance occur with decreased renal function. Other age-related changes that may affect diabetes and its management include hypertension, decreased bowel motility, and decreased thirst.

Which type of insulin acts most quickly? a) Glargine b) NPH c) Regular d) Lispro

b) NPH

An older adult patient is in the hospital being treated for sepsis related to a urinary tract infection. The patient has started to have an altered sense of awareness, profound dehydration, and hypotension. What does the nurse suspect the patient is experiencing? a. systemic inflammatory response syndrome b. hyperglycemic hyperosmolar syndrome c. multiple-organ dysfunction syndrome d. Diabetic ketoacidosis

b. hyperglycemic hyperosmolar syndrome

Every morning, a client with type 1 diabetes receives 15 units of Humulin 70/30. What does this type of insulin contain? a) 70 units of regular insulin and 30 units of NPH insulin b) 70% regular insulin and 30% NPH insulin c) 70% NPH insulin and 30% regular insulin d) 70 units of neutral protamine Hagedorn (NPH) insulin and 30 units of regular insulin

c) 70% NPH insulin and 30% regular insulin

While conducting a physical examination of a client, which of the following skin findings would alert the nurse to the possibility of liver problems? Select all that apply.

Jaundice Petechiae Ecchymoses

Which instruction about insulin administration should a nurse give to a client? A. "Shake the vials before withdrawing the insulin." B. "Always follow the same order when drawing the different insulins into the syringe." C. "Discard the intermediate-acting insulin if it appears cloudy." D. "Store unopened vials of insulin in the freezer at temperatures well below freezing."

"Always follow the same order when drawing different insulins into the syringe." The nurse should instruct the client to always follow the same order when drawing the different insulins into the syringe. Insulin should never be shaken because the resulting froth prevents withdrawal of an accurate dose and may damage the insulin protein molecules. Insulin should never be frozen because the insulin protein molecules may be damaged. The client doesn't need to discard intermediate-acting insulin if it's cloudy; this finding is normal.

The nurse is educating a patient about the benefits of fruit versus fruit juice in the diabetic diet. The patient states, "What difference does it make if you drink the juice or eat the fruit? It is all the same." What is the best response by the nurse? A. "Eating the fruit is more satisfying than drinking the juice. You will get full faster." B. "Eating the fruit instead of drinking juice decreases the glycemic index by slowing absorption." C. "Eating the fruit will give you more vitamins and minerals than the juice will." D. "The fruit has less sugar than the juice."

"Eating the fruit instead of drinking juice decreases the glycemic index by slowing absorption." Eating whole fruit instead of drinking juice decreases the glycemic index, because fiber in the fruit slows absorption.

After teaching a client with type 1 diabetes who is scheduled to undergo an islet cell transplant, which client statement indicates successful teaching? a. "This transplant will provide me with a cure for my diabetes." b. "I will receive a whole organ with extra cells to produce insulin." c. "They'll need to create a connection from the pancreas to allow enzymes to drain." d. "I might need insulin later on but probably not as much or as often."

"I might need insulin later on but probably not as much or as often." Transplanted islet cells tend to lose their ability to function over time, and approximately 70% of recipients resume insulin administration in 2 years. However, the amount of insulin and the frequency of its administration are reduced because of improved control of blood glucose levels. Thus, this type of transplant doesn't cure diabetes. It requires the use of two human pancreases to obtain sufficient numbers of islet cells for transplantation. A whole organ transplant requires a means for exocrine enzyme drainage and venous absorption of insulin.

A client with type 2 diabetes has recently been prescribed acarbose, and the nurse is explaining how to take this medication. The teaching is determined to be effective based on which statement by the client?

"I will take this medication in the morning, with my first bite of breakfast." Alpha-glucosidase inhibitors such as acarbose and miglitol, delay absorption of complex carbohydrates in the intestine and slow entry of glucose into systemic circulation. They must be taken with the first bite of food to be effective.

Which statement indicates that a client with diabetes mellitus understands proper foot care?

"I'll wear cotton socks with well-fitting shoes."

A nurse is preparing to administer insulin to a child who's just been diagnosed with type 1 diabetes. When the child's mother stops the nurse in the hall, she's crying and anxious to talk about her son's condition. The nurse's best response is: "I can't talk now. I have to give your son his insulin as soon as possible." "I'm going to give your son some insulin. Then I'll be happy to talk with you." "Everything will be just fine. I'll be back in a minute and then we can talk." "If you'll wait in your son's room, the physician will talk with you as soon as he's free."

"I'm going to give your son some insulin. Then I'll be happy to talk with you." Attending to the mother's needs is a critical part of caring for a sick child. In this case however, administering insulin in a prompt manner supersedes the mother's needs. By informing the mother that she's going to administer the insulin and will then make time to talk with her, the nurse recognizes the mother's needs as legitimate. She provides a reasonable response while attending to the priority of administering insulin as soon as possible. Telling the mother that she can't talk with her or telling her to wait for the physician could increase the mother's fear and anxiety. The nurse shouldn't tell the mother that everything will be fine; the nurse doesn't know that everything will be fine.

A client is evaluated for type 1 diabetes. Which client comment correlates best with this disorder? A. "I'm thirsty all the time. I just can't get enough to drink." B. "I notice pain when I urinate." C. "It seems like I have no appetite. I have to make myself eat." D. "I have a cough and cold that just won't go away."

"I'm thirsty all the time. I just can't get enough to drink." The nurse should instruct a client with diabetes mellitus to check his blood glucose levels every 3 to 4 hours and take insulin or an oral antidiabetic agent as usual, even when he's sick. If the client's blood glucose level rises above 300 mg/dl, he should call his physician immediately. If the client is unable to follow the regular meal plan because of nausea, he should substitute soft foods, such as gelatin, soup, and custard.

A client with type 1 diabetes has a highly elevated glycosylated hemoglobin (Hb) test result. In discussing the result with the client, the nurse is most accurate in stating:

"It tells us about your sugar control for the last 3 months." Explanation: The nurse is providing accurate information to the client when she states that the glycosylated Hb test provides an objective measure of glycemic control over a 3-month period. The test helps identify trends or practices that impair glycemic control, and it doesn't require a fasting period before blood is drawn. The nurse can't conclude that the result occurs from poor dietary management or inadequate insulin coverage.

A client newly diagnosed with diabetes mellitus asks why he needs ketone testing when the disease affects his blood glucose levels. How should the nurse respond? A. "Ketones can damage your kidneys and eyes." B. "Ketones will tell us if your body is using other tissues for energy." C. "Ketones help the physician determine how serious your diabetes is." D. "The spleen releases ketones when your body can't use glucose."

"Ketones will tell us if your body is using other tissues for energy." The nurse should tell the client that ketones are a byproduct of fat metabolism and that ketone testing can determine whether the body is breaking down fat to use for energy. The spleen doesn't release ketones when the body can't use glucose. Although ketones can damage the eyes and kidneys and help the physician evaluate the severity of a client's diabetes, these responses by the nurse are incomplete.

A client with diabetes mellitus must learn how to self-administer insulin. The physician has ordered 10 units of U-100 regular insulin and 35 units of U-100 isophane insulin suspension (NPH) to be taken before breakfast. When teaching the client how to select and rotate insulin injection sites, the nurse should provide which instruction? A. "Rotate injection sites within the same anatomic region, not among different regions." B. "Administer insulin into sites above muscles that you plan to exercise heavily later that day." C. "Inject insulin into healthy tissue with large blood vessels and nerves." D. "Administer insulin into areas of scar tissue or hypertrophy whenever possible."

"Rotate injection sites within the same anatomic region, not among different regions." The nurse should instruct the client to rotate injection sites within the same anatomic region. Rotating sites among different regions may cause excessive day-to-day variations in the blood glucose level; also, insulin absorption differs from one region to the next. Insulin should be injected only into healthy tissue lacking large blood vessels, nerves, or scar tissue or other deviations. Injecting insulin into areas of hypertrophy may delay absorption. The client shouldn't inject insulin into areas of lipodystrophy (such as hypertrophy or atrophy); to prevent lipodystrophy, the client should rotate injection sites systematically. Exercise speeds drug absorption, so the client shouldn't inject insulin into sites above muscles that will be exercised heavily.

A client is receiving insulin lispro at 7:30 AM. The nurse ensures that the client has breakfast by which time? a. 7:45 AM b. 8:00 AM c. 8:15 AM d. 8:30 AM

7:45 AM Insulin lispro has an onset of 5 to 15 minutes. Therefore, the nurse would need to ensure that the client has his breakfast by 7:45 AM at the latest. Otherwise, the client may experience hypoglycemia.

Health teaching for a patient with diabetes who is prescribed Humulin N, an intermediate NPH insulin, would include which of the following advice? a) "Your insulin will begin to act in 15 minutes." b) "Your insulin will last 8 hours, and you will need to take it three times a day." c) "You should take your insulin after you eat breakfast and dinner." d) "You should expect your insulin to reach its peak effectiveness by 12 noon if you take it at 8:00 AM."

"You should take your insulin after you eat breakfast and dinner."

A client has just been diagnosed with type 1 diabetes. When teaching the client and family how diet and exercise affect insulin requirements, the nurse should include which guideline? A. "You'll need more insulin when you exercise or decrease your food intake." B. "You'll need less insulin when you exercise or reduce your food intake." C. "You'll need more insulin when you exercise or increase your food intake." D. "You'll need less insulin when you increase your food intake."

"You'll need less insulin when you exercise or reduce your food intake." The nurse should advise the client that exercise, reduced food intake, hypothyroidism, and certain medications decrease insulin requirements. Growth, pregnancy, greater food intake, stress, surgery, infection, illness, increased insulin antibodies, and certain medications increase insulin requirements.

The nurse is developing a care plan for the client diagnosed with type 1 diabetes. The nurse identifies the problem "high risk for hyperglycemia related to noncompliance with the medication regimen." Which statement is an appropriate short-term goal for the client? 1. The client will have a blood glucose level between 90 and 140 mg/dL. 2. The client will demonstrate appropriate insulin injection technique. 3. The nurse will monitor the client's blood glucose levels four (4) times a day. 4. The client will maintain normal kidney function with 30-mL/hr urine output.

1. The client will have a blood glucose level between 90 and 140 mg/dL.

A client with diabetes mellitus has good glucose level of 40 mg/dl. Which rapidly absorbed carbohydrate would be most effective?

1/2 cup fruit juice or regular soft drink

A physician orders blood glucose levels every 4 hours for a 4-year-old child with brittle type 1 diabetes. The parents are worried that drawing so much blood will traumatize their child. How can the nurse best reassure the parents? A. "Your child will need less blood work as his glucose levels stabilize." B. "Our laboratory technicians use tiny needles and they're really good with children." C. "Your child is young and will soon forget this experience." D. "I'll see if the physician can reduce the number of blood draws."

"Your child will need less blood work as his glucose levels stabilize." Telling the parents that the number of blood draws will decrease as their child's glucose levels stabilize engages them in the learning process and gives them hope that the present discomfort will end as the child's condition improves. Telling the parents that their child won't remember the experience disregards their concerns and anxiety. The nurse shouldn't offer to ask the physician if he can reduce the number of blood draws; the physician needs the laboratory results to monitor the child's condition properly. Although telling the parents that the laboratory technicians are gentle and use tiny needles may be reassuring, it isn't the most appropriate response.

A physician orders blood glucose levels every 4 hours for a 4-year-old child with brittle type 1 diabetes. The parents are worried that drawing so much blood will traumatize their child. How can the nurse best reassure the parents? "I'll see if the physician can reduce the number of blood draws." "Our laboratory technicians use tiny needles and they're really good with children." "Your child is young and will soon forget this experience." "Your child will need less blood work as his glucose levels stabilize."

"Your child will need less blood work as his glucose levels stabilize." Telling the parents that the number of blood draws will decrease as their child's glucose levels stabilize engages them in the learning process and gives them hope that the present discomfort will end as the child's condition improves. Telling the parents that their child won't remember the experience disregards their concerns and anxiety. The nurse shouldn't offer to ask the physician if he can reduce the number of blood draws; the physician needs the laboratory results to monitor the child's condition properly. Although telling the parents that the laboratory technicians are gentle and use tiny needles may be reassuring, it isn't the most appropriate response.

A diabetes educator is teaching a client about type 2 diabetes. The educator recognize that the client understands the primary treatment for type 2 diabetes when the client states what?

"i will make sure to follow the weight loss plan designed by the dietitian"

A diabetic educator is discussing "sick day rules" with a newly diagnosed type 1 diabetic. The educator is aware that the client will require further teaching when the client states that?

"i will not take my insulin on the days when I am sick, but I will certainly check my blood sugar every 2 hours"

A nurse is caring for an elderly patient who has type 2 diabetes mellitus. She suspects that the patient is exhibiting symptoms of diabetic ketoacidosis (DKA) instead of hyperglycemic hyperosmolar nonketotic syndrome (HHNS). Which of the following are indicators of a diagnosis of DKA? Select all that apply.

- Blood glucose level of 280 mg/dL - Arterial pH of 7 - Plasma bicarbonate level of 13 mEq/L

The home health nurse is completing the admission assessment for a 76-year-old client diagnosed with type 2 diabetes controlled with 70/30 insulin. Which intervention should be included in the plan of care? 1. Assess the client's ability to read small print. 2. Monitor the client's serum PT level. 3. Teach the client how to perform a hemoglobin A1c test daily. 4. Instruct the client to check the feet weekly.

1. Assess the client's ability to read small print.

The nurse administered 28 units of Humulin N, an intermediate-acting insulin, to a client diagnosed with type 1 diabetes at 1600. Which intervention should the nurse implement? 1. Ensure the client eats the bedtime snack. 2. Determine how much food the client ate at lunch. 3. Perform a glucometer reading at 0700. 4. Offer the client protein after administering insulin.

1. Ensure the client eats the bedtime snack.

The elderly client is admitted to the intensive care department diagnosed with severe HHNS. Which collaborative intervention should the nurse include in the plan of care? 1. Infuse 0.9% normal saline intravenously. 2. Administer intermediate-acting insulin. 3. Perform blood glucometer checks daily. 4. Monitor arterial blood gas results.

1. Infuse 0.9% normal saline intravenously

The diabetic educator is teaching a class on diabetes type 1 and is discussing sick-day rules. Which interventions should the diabetes educator include in the discussion? Select all that apply. 1. Take diabetic medication even if unable to eat the client's normal diabetic diet. 2. If unable to eat, drink liquids equal to the client's normal caloric intake. 3. It is not necessary to notify the health-care provider if ketones are in the urine. 4. Test blood glucose levels and test urine ketones once a day and keep a record. 5. Call the health-care provider if glucose levels are higher than 180 mg/dL.

1. Take diabetic medication even if unable to eat the client's normal diabetic diet. 2. If unable to eat, drink liquids equal to the client's normal caloric intake. 5. Call the health-care provider if glucose levels are higher than 180 mg/dL.

A client with diabetes mellitus has a blood glucose level of 40 mg/dL. Which rapidly absorbed carbohydrate would be most effective?

1/2 cup fruit juice or regular soft drink : In a client with hypoglycemia, the nurse uses the rule of 15: give 15 g of rapidly absorbed carbohydrate, wait 15 minutes, recheck the blood sugar, and administer another 15 g of glucose if the blood sugar is not above 70 mg/dL. One-half cup fruit juice or regular soft drink is equivalent to the recommended 15 g of rapidly absorbed carbohydrate. Eight ounces of skim milk is equivalent to the recommended 15 g of rapidly absorbed carbohydrate. One tablespoon of honey or syrup is equivalent to the recommended 15 g of rapidly absorbed carbohydrate. Six to eight LifeSavers candies is equivalent to the recommended 15 g of rapidly absorbed carbohydrate.

An agitated, confused client arrives in the emergency department. The client's history includes type 1 diabetes, hypertension, and angina pectoris. Assessment reveals pallor, diaphoresis, headache, and intense hunger. A stat blood glucose sample measures 42 mg/dl, and the client is treated for an acute hypoglycemic reaction. After recovery, the nurse teaches the client to treat hypoglycemia by ingesting: A. 10 to 15 g of a simple carbohydrate. B. 25 to 30 g of a simple carbohydrate. C. 2 to 5 g of a simple carbohydrate. D. 18 to 20 g of a simple carbohydrate.

10 to 15 g of a simple carbohydrate To reverse hypoglycemia, the American Diabetes Association recommends ingesting 10 to 15 g of a simple carbohydrate, such as three to five pieces of hard candy, two to three packets of sugar (4 to 6 tsp), or 4 oz of fruit juice. Then the client should check his blood glucose after 15 minutes. If necessary, this treatment may be repeated in 15 minutes. Ingesting only 2 to 5 g of a simple carbohydrate may not raise the blood glucose level sufficiently. Ingesting more than 15 g may raise it above normal, causing hyperglycemia.

The nurse is administering lispro insulin. Based on the onset of action, how long before breakfast should the nurse administer the injection? A. 10 to 15 minutes B. 30 to 40 minutes C. 1 to 2 hours D. 3 hours

10 to 15 minutes The onset of action of rapid-acting lispro insulin is within 10 to 15 minutes. It is used to rapidly reduce the glucose level.

Once digested, what percentage of carbohydrates is converted to glucose?

100 Explanation: Once digested, 100% of carbohydrates are converted to glucose. However, approximately 40% of protein foods are also converted to glucose, but this has minimal effect on blood glucose levels.

A nurse is preparing a continuous insulin infusion for a child with diabetic ketoacidosis and a blood glucose level of 800 mg/dl. Which solution is the most appropriate at the beginning of therapy? 100 units of NPH insulin in dextrose 5% in water 100 units of regular insulin in dextrose 5% in water 100 units of neutral protamine Hagedorn (NPH) insulin in normal saline solution 100 units of regular insulin in normal saline solution

100 units of regular insulin in normal saline solution Continuous insulin infusions use only short-acting regular insulin. Insulin is added to normal saline solution and administered until the client's blood glucose level falls. Further along in the therapy, a dextrose solution is administered to prevent hypoglycemia.

A nurse is aware that insulin secretion increases 3 to 5 minutes after a meal and then returns to baseline. If a patient ate breakfast at 7:30 AM, the nurse would expect a baseline level by:

10:30 AM

The Lowest fasting plasma glucose level suggestive of a diagnosis fo diabetes is:

115 mg/dl

A medical nurse is caring for a client with type 1 diabetes. The client's medication administration record includes the administration of regular insulin three times daily. Knowing that the client's lunch tray will arrive at 11:45 AM, when should the nurse administer the client's insulin?

11:15 AM

A 6 months' pregnant patient was evaluated for gestational diabetes mellitus. The doctor considered prescribing insulin based on the serum glucose result of: A. 120 mg/dL, 1 hour postprandial. B. 138 mg/dL, 2 hours postprandial. C. 90 mg/dL before meals. D. 80 mg/dL, 1 hour postprandial.

138 mg/dL, 2 hours postprandial The goals for a 2-hour, postprandial blood glucose level are less than 120 mg/dL in a patient who might develop gestational diabetes.

A nurse is teaching a client with diabetes mellitus about self-management of his condition. The nurse should instruct the client to administer 1 unit of insulin for every: A. 10 g of carbohydrates. B. 15 g of carbohydrates. C. 20 g of carbohydrates. D. 25 g of carbohydrates.

15 g of carbohydrates The nurse should instruct the client to administer 1 unit of insulin for every 15 g of carbohydrates.

A nurse is caring for a client with type 1 diabetes who exhibits confusion, light-headedness, and aberrant behavior. The client is conscious. The nurse should first administer:

15 to 20 g of a fast-acting carbohydrate such as orange juice. Explanation: This client is experiencing hypoglycemia. Because the client is conscious, the nurse should first administer a fast-acting carbohydrate, such as orange juice, hard candy, or honey. If the client has lost consciousness, the nurse should administer I.M. or subcutaneous glucagon or an I.V. bolus of dextrose 50%. The nurse shouldn't administer insulin to a client who's hypoglycemic; this action will further compromise the client's condition

The nurse at a freestanding health care clinic is caring for a 56-year-old male client who is homeless and is a type 2 diabetic controlled with insulin. Which action is an example of client advocacy? 1. Ask the client if he has somewhere he can go and live. 2. Arrange for someone to give him insulin at a local homeless shelter. 3. Notify Adult Protective Services about the client's situation. 4. Ask the HCP to take the client off insulin because he is homeless.

2. Arrange for someone to give him insulin at a local homeless shelter.

The client with type 2 diabetes controlled with biguanide oral diabetic medication is scheduled for a computed tomography (CT) scan with contrast of the abdomen to evaluate pancreatic function. Which intervention should the nurse implement? 1. Provide a high-fat diet 24 hours prior to test. 2. Hold the biguanide medication for 48 hours prior to test. 3. Obtain an informed consent form for the test. 4. Administer pancreatic enzymes prior to the test.

2. Hold the biguanide medication for 48 hours prior to test.

The UAP on the medical floor tells the nurse the client diagnosed with DKA wants something else to eat for lunch. Which intervention should the nurse implement? 1. Instruct the UAP to get the client additional food. 2. Notify the dietitian about the client's request. 3. Request the HCP increase the client's caloric intake. 4. Tell the UAP the client cannot have anything else.

2. Notify the dietitian about the client's request.

Which assessment data indicate the client diagnosed with diabetic ketoacidosis is responding to the medical treatment? 1. The client has tented skin turgor and dry mucous membranes. 2. The client is alert and oriented to date, time, and place. 3. The client's ABG results are pH 7.29, PaCO2 44, HCO3 15. 4. The client's serum potassium level is 3.3 mEq/L.

2. The client is alert and oriented to date, time, and place.

Glycosylated hemoglobin reflects blood glucose concentrations over which period of time?

3 months

The client received 10 units of Humulin R, a fast-acting insulin, at 0700. At 1030 the unlicensed assistive personnel (UAP) tells the nurse the client has a headache and is really acting "funny." Which intervention should the nurse implement first? 1. Instruct the UAP to obtain the blood glucose level. 2. Have the client drink eight (8) ounces of orange juice. 3. Go to the client's room and assess the client for hypoglycemia. 4. Prepare to administer one (1) ampule 50% dextrose intravenously.

3. Go to the client's room and assess the client for hypoglycemia.

The nurse is discussing ways to prevent diabetic ketoacidosis with the client diagnosed with type 1 diabetes. Which instruction is most important to discuss with the client? 1. Refer the client to the American Diabetes Association. 2. Do not take any over-the-counter medications. 3. Take the prescribed insulin even when unable to eat because of illness. 4. Explain the need to get the annual flu and pneumonia vaccines.

3. Take the prescribed insulin even when unable to eat because of illness.

The nurse is assessing the feet of a client with long-term type 2 diabetes. Which assessment data warrant immediate intervention by the nurse? 1. The client has crumbling toenails. 2. The client has athlete's foot. 3. The client has a necrotic big toe. 4. The client has thickened toenails.

3. The client has a necrotic big toe.

The client diagnosed with type 1 diabetes has a glycosylated hemoglobin (A1c) of 8.1%. Which interpretation should the nurse make based on this result? 1. This result is below normal levels. 2. This result is within acceptable levels. 3. This result is above recommended levels. 4. This result is dangerously high.

3. This result is above recommended levels.

The client diagnosed with HHNS was admitted yesterday with a blood glucose level of 780 mg/dL. The client's blood glucose level is now 300 mg/dL. Which intervention should the nurse implement? 1. Increase the regular insulin IV drip. 2. Check the client's urine for ketones. 3. Provide the client with a therapeutic diabetic meal. 4. Notify the HCP to obtain an order to decrease insulin.

4. Notify the HCP to obtain an order to decrease insulin.

The nurse is discussing the importance of exercising with a client diagnosed with type 2 diabetes whose diabetes is well controlled with diet and exercise. Which information should the nurse include in the teaching about diabetes? 1. Eat a simple carbohydrate snack before exercising. 2. Carry peanut butter crackers when exercising. 3. Encourage the client to walk 20 minutes three (3) times a week. 4. Perform warmup and cool-down exercises.

4. Perform warmup and cool-down exercises.

Which arterial blood gas results should the nurse expect in the client diagnosed with diabetic ketoacidosis? 1. pH 7.34, PaO2 99, PaCO2 48, HCO3 24. 2. pH 7.38, PaO2 95, PaCO2 40, HCO3 22. 3. pH 7.46, PaO2 85, PaCO2 30, HCO3 26. 4. pH 7.30, PaO2 90, PaCO2 30, HCO3 18.

4. pH 7.30, PaO2 90, PaCO2 30, HCO3 18.

A client with diabetes comes to the clinic for a follow-up visit. The nurse reviews the client's glycosylated hemoglobin test results. Which result would indicate to the nurse that the client's blood glucose level has been well-controlled? A. 7.5 % B. 8.0% C. 6.5% D. 8.5%

6.5% Normally the level of glycosylated hemoglobin is less than 7%. Thus a level of 6.5% would indicate that the client's blood glucose level is well-controlled. According to the American Diabetes Association, a glycosylated hemoglobin of 7% is equivalent to an average blood glucose level of 150 mg/dL. Thus, a level of 7.5% would indicate less control. Amount of 8% or greater indicate that control of the client's blood glucose level has been inadequate during the previous 2 to 3 months.

A client with type 1 diabetes is scheduled to receive 30 units of 70/30 insulin. There is no 70/30 insulin available. As a substitution, the nurse may give the client: a) 20 units regular insulin and 10 units NPH. b) 21 units regular insulin and 9 units NPH. c) 10 units regular insulin and 20 units NPH. d) 9 units regular insulin and 21 units neutral protamine Hagedorn (NPH).

9 units regular insulin and 21 units neutral protamine Hagedorn (NPH). A 70/30 insulin preparation is 70% NPH and 30% regular insulin. Therefore, a correct substitution requires mixing 21 units of NPH and 9 units of regular insulin. The other choices are incorrect dosages for the ordered insulin.

Which statement is true regarding gestational diabetes?

A glucose challenge test should be performed between 24 to 28 weeks.

A hospitalized client is found to be comatose and hypoglycemic with a blood sugar of 50 mg/dL. Which of the following would the nurse do first? A. Check the client's urine for the presence of sugar and acetone. B. Administer 50% glucose intravenously. C. Encourage the client to drink orange juice with added sugar. D. Infuse 1000 mL D5W over a 12-hour period.

Administer 50% glucose intravenously. The unconscious, hypoglycemic client needs immediate treatment with IV glucose. If the client does not respond quickly and the blood glucose level continues to be low, glucagon, a hormone that stimulates the liver to release glycogen, or 20 to 50 mL of 50% glucose is prescribed for IV administration. A dose of 1,000 mL D5W over a 12-hour period indicates a lower strength of glucose and a slow administration rate. Checking the client's urine for the presence of sugar and acetone is incorrect because a blood sample is easier to collect and the blood test is more specific and reliable. An unconscious client cannot be given a drink. In such a case glucose gel may be applied in the buccal cavity of the mouth.

A client with diabetic ketoacidosis has been brought into the ED. Which intervention is not a goal in the initial medical treatment of diabetic ketoacidosis?

Administer glucose.

A client with type 1 diabetes must undergo bowel resection in the morning. How should the nurse proceed while caring for him on the morning of surgery?

Administer half of the client's typical morning insulin dose as ordered. If the nurse administers the client's normal daily dose of insulin while he's on nothing-by-mouth status before surgery, he'll experience hypoglycemia. Therefore, the nurse should administer half the daily insulin dose as ordered. Oral antidiabetic agents aren't effective for type 1 diabetes. I.V. insulin infusions aren't necessary to manage blood glucose levels in clients undergoing routine surgery.

A client with type 1 diabetes presents with a decreased level of consciousness and a fingerstick glucose level of 39 mg/dl. His family reports that he has been skipping meals in an effort to lose weight. Which nursing intervention is most appropriate? A. Administering 1 ampule of 50% dextrose solution, per physician's order B. Administering a 500-ml bolus of normal saline solution C. Inserting a feeding tube and providing tube feedings D. Observing the client for 1 hour, then rechecking the fingerstick glucose level

Administering 1 ampule of 50% dextrose solution, per physician's order The nurse should administer 50% dextrose solution to restore the client's physiological integrity. Feeding through a feeding tube isn't appropriate for this client. A bolus of normal saline solution doesn't provide the client with the much-needed glucose. Observing the client for 1 hour delays treatment. The client's blood glucose level could drop further during this time, placing him at risk for irreversible brain damage.

Management of a patient with ascites includes nutritional modifications and diuretic therapy. Which of the following interventions would a nurse expect to be part of patient care? Select all that apply.

Aldactone, an aldosterone-blocking agent would be used. Daily salt intake would be restricted to 2 grams or less. The diuretic will be held if the serum sodium level decreases to <134 m Eq/L.

The client is admitted to the ICU diagnosed with DKA. Which interventions should the nurse implement? Select all that apply. 1. Maintain adequate ventilation. 2. Assess fluid volume status. 3. Administer intravenous potassium. 4. Check for urinary ketones. 5. Monitor intake and output.

All of them 1. Maintain adequate ventilation. 2. Assess fluid volume status. 3. Administer intravenous potassium. 4. Check for urinary ketones. 5. Monitor intake and output.

A client has been diagnosed with prediabetes and discusses treatment strategies with the nurse. What can be the consequences of untreated prediabetes? A. type 2 diabetes B. cardiac disease C. CVA D. All options are correct.

All options are correct The NIDDK has developed criteria that identify people with prediabetes, which can lead to type 2 diabetes, heart disease, and stroke.

Which would be included in the teaching plan for a client diagnosed with diabetes mellitus?

An elevated blood glucose concentration contributes to complications of diabetes, such as diminished vision.

An older adult client is diagnosed with acute pancreatitis. Using what the nurse understands about gerontologic considerations related to acute pancreatitis, what concept does the nurse understand? Select all that apply.

As the client ages, there is an increased mortality rate for acute pancreatitis. As the client ages, there is an increased risk for the development of multiple organ dysfunction syndrome. As the client ages, the pattern of complications related to acute pancreatitis changes.

A nurse has been caring for a client newly diagnosed with diabetes mellitus. The client is overwhelmed by what he's facing and not sure he can handle giving himself insulin. This client has been discharged and the charge nurse is insisting the nurse hurry because she needs the space for clients being admitted. How should the nurse handle the situation? a) Tell the charge nurse she doesn't believe this client will be safe and refuse to rush. b) Ask the physician for a referral for a diabetes nurse-educator to see the client before discharge. c) Suggest the client find a supportive friend or family member to assist in his care. d) Ask the physician to delay the discharge because the client requires further teaching.

Ask the physician to delay the discharge because the client requires further teaching. The nurse's primary concern should be the safety of the client after discharge. She should provide succinct information to the physician concerning the client's needs, express her concern about ensuring the client's safety, and ask the physician to delay the client's discharge. The nurse shouldn't suggest that the client rely on a friend or family member because she doesn't know if a friend or family member will be available to help. Refusing to rush and telling the charge nurse she isn't sure the client will be safe demonstrate appropriate intentions, but these actions don't alleviate the pressure to discharge the client. Asking a physician to refer the client to a diabetic nurse-educator addresses the client's needs, but isn't the best response because there's no guarantee a diabetic nurse-educator will be available on such short notice.

A client is admitted to the health care center with abdominal pain, nausea, and vomiting. The medical reports indicate a history of type 1 diabetes. The nurse suspects the client's symptoms to be those of diabetic ketoacidosis (DKA). Which action will help the nurse confirm the diagnosis?

Assess the client's breath odor : DKA is commonly preceded by a day or more of polyuria, polydipsia, nausea, vomiting, and fatigue, with eventual stupor and coma if not treated. The breath has a characteristic fruity odor due to the presence of ketoacids. Checking the client's breath will help the nurse confirm the diagnosis.

A client with type 1 diabetes mellitus is seeing the nurse to review foot care. What would be a priority instruction for the nurse to give the client?

Avoid hot wet bottles and heating pads

A child is brought into the emergency department with vomiting, drowsiness, and blowing respirations. The father reports that the symptoms have been progressing throughout the day. The nurse suspects diabetic ketoacidosis (DKA). Which action should the nurse take first in the management of DKA?

Begin fluid replacements Management of DKA is aimed at correcting dehydration, electrolyte loss, and acidosis before correcting the hypoglycemia with insulin.

A nurse expects to note an elevated serum glucose level in a client with hyperosmolar hyperglycemic nonketotic syndrome (HHNS). Which other laboratory finding should the nurse anticipate?

Below-normal serum potassium level A client with HHNS has an overall body deficit of potassium resulting from diuresis, which occurs secondary to the hyperosmolar, hyperglycemic state caused by the relative insulin deficiency. An elevated serum acetone level and serum ketone bodies are characteristic of diabetic ketoacidosis. Metabolic acidosis, not serum alkalosis, may occur in HHNS.

Insulin is secreted by which of the following types of cells?

Beta cells Explanation: Insulin is secreted by the beta cells, in the islets of Langerhans of the pancreas. In diabetes, cells may stop responding to insulin, or the pancreas may decrease insulin secretion or stop insulin production completely. Melanocytes are what give the skin its pigment. Neural cells transmit impulses in the brain and spinal cord. Basal cells are a type of skin cell.

Which clinical characteristic is associated with type 2 diabetes (previously referred to as non-insulin-dependent diabetes mellitus)? A. Clients demonstrate islet cell antibodies B. Blood glucose can be controlled through diet and exercise C. Client is usually thin at diagnosis D. Client is prone to ketosis

Blood glucose can be controlled through diet and exercise Oral hypoglycemic agents may improve blood glucose concentrations if dietary modification and exercise are unsuccessful. Individuals with type 2 diabetes are usually obese at diagnosis. Individuals with type 2 diabetes rarely demonstrate ketosis, except with stress or infection. Individuals with type 2 diabetes do not demonstrate islet cell antibodies.

The nurse is reviewing the initial laboratory test results of a client diagnosed with DKA. Which of the following would the nurse expect to find? a) Blood pH of 6.9 b) Blood glucose level of 250 mg/dL c) Serum bicarbonate of 19 mEq/L d) PaCO2 of 40 mm Hg

Blood pH of 6.9 Explanation: With DKA, blood glucose levels are elevated to 300 to 1000 mg/dL or more. Urine contains glucose and ketones. The blood pH ranges from 6.8 to 7.3. The serum bicarbonate level is decreased to levels from 0 to 15 mEq/L. The compensatory breathing pattern can lower the partial pressure of carbon dioxide in arterial blood (PaCO2) to levels of 10 to 30 mm Hg.

The nurse practitioner worked with a registered dietician to customize a 1,800-calorie diabetic diet for a 53-year-old man with special dietary needs. Which of the following percent distribution of calories should be provided?

Carbohydrates 50%, fat 25%, and protein 20%

A client with diabetic ketoacidosis was admitted to the intensive care unit 4 hours ago and has these laboratory results: blood glucose level 450 mg/dl, serum potassium level 2.5 mEq/L, serum sodium level 140 mEq/L, and urine specific gravity 1.025. The client has two I.V. lines in place with normal saline solution infusing through both. Over the past 4 hours, his total urine output has been 50 ml. Which physician order should the nurse question?

Change the second I.V. solution to dextrose 5% in water. The nurse should question the physician's order to change the second I.V. solution to dextrose 5% in water. The client should receive normal saline solution through the second I.V. site until his blood glucose level reaches 250 mg/dl. The client should receive a fluid bolus of 500 ml of normal saline solution. The client's urine output is low and his specific gravity is high, which reveals dehydration. The nurse should expect to hold the insulin infusion for 30 minutes until the potassium replacement has been initiated. Insulin administration causes potassium to enter the cells, which further lowers the serum potassium level. Further lowering the serum potassium level places the client at risk for life-threatening cardiac arrhythmias.

Which factors will cause hypoglycemia in a client with diabetes? Select all that apply.

Client has not consumed food and continues to take insulin or oral antidiabetic medications. Client has not consumed sufficient calories. Client has been exercising more than usual.

A visiting nurse is setting up an insulin schedule for an older adult who has diabetes mellitus. What should the nurse consider when determine the dosing time?

Clients eating and sleeping habits

Which of the following would be inconsistent as a cause of DKA?

Competency in injecting insulin

A nurse educates a group of clients with diabetes mellitus on the prevention of diabetic nephropathy. Which of the following suggestions would be most important? A. Control blood glucose levels. B. Eat a high-fiber diet. C. Drink plenty of fluids. D. Take the antidiabetic drugs regularly.

Control blood glucose levels. Controlling blood glucose levels and any hypertension can prevent or delay the development of diabetic nephropathy. Drinking plenty of fluids does not prevent diabetic nephropathy. Taking antidiabetic drugs regularly may help to control blood glucose levels, but it is the control of these levels that is most important. A high-fiber diet is unrelated to the development of diabetic nephropathy.

A client is diagnosed with diabetes mellitus. Which assessment finding best supports a nursing diagnosis of Ineffective coping related to diabetes mellitus? A. Recent weight gain of 20 lb (9.1 kg) B. Failure to monitor blood glucose levels C. Crying whenever diabetes is mentioned D. Skipping insulin doses during illness

Crying whenever diabetes is mentioned A client who cries whenever diabetes is mentioned is demonstrating ineffective coping. A recent weight gain and failure to monitor blood glucose levels would support a nursing diagnosis of Noncompliance: Failure to adhere to therapeutic regimen. Skipping insulin doses during illness would support a nursing diagnosis of Deficient knowledge related to treatment of diabetes mellitus.

A client with long-standing type 1 diabetes is admitted to the hospital with unstable angina pectoris. After the client's condition stabilizes, the nurse evaluates the diabetes management regimen. The nurse learns that the client sees the physician every 4 weeks, injects insulin after breakfast and dinner, and measures blood glucose before breakfast and at bedtime. Consequently, the nurse should formulate a nursing diagnosis of: A. Deficient knowledge (treatment regimen). B. Impaired adjustment. C. Health-seeking behaviors (diabetes control). D. Defensive coping.

Deficient knowledge (treatment regimen) The client should inject insulin before, not after, breakfast and dinner — 30 minutes before breakfast for the a.m. dose and 30 minutes before dinner for the p.m. dose. Therefore, the client has a knowledge deficit regarding when to administer insulin. By taking insulin, measuring blood glucose levels, and seeing the physician regularly, the client has demonstrated the ability and willingness to modify his lifestyle as needed to manage the disease. This behavior eliminates the nursing diagnoses of Impaired adjustment and Defensive coping. Because the nurse, not the client, questioned the client's health practices related to diabetes management, the nursing diagnosis of Health-seeking behaviors isn't warranted.

An obese Hispanic client, age 65, is diagnosed with type 2 diabetes. Which statement about diabetes mellitus is true? A. Nearly two-thirds of clients with diabetes mellitus are older than age 60. B. Diabetes mellitus is more common in Hispanics and Blacks than in Whites. C. Approximately one-half of the clients diagnosed with type 2 diabetes are obese. D. Type 2 diabetes mellitus is less common than type 1 diabetes mellitus.

Diabetes mellitus is more common in Hispanics and Blacks than in Whites Diabetes mellitus is more common in Hispanics and Blacks than in Whites. Only about one-third of clients with diabetes mellitus are older than age 60 and 85% to 90% have type 2. At least 80% of clients diagnosed with type 2 diabetes mellitus are obese.

A nurse is preparing a client with type 1 diabetes for discharge. The client can care for himself; however, he's had a problem with unstable blood glucose levels in the past. Based on the client's history, he should be referred to which health care worker? a) Social worker b) Home health nurse c) Psychiatrist d) Dietitian

Dietitian The client should be referred to a dietitian, who will help him gain better control of his blood glucose levels. The client can care for himself, so a home health agency isn't necessary. The client shows no signs of needing a psychiatric referral, and referring the client to a psychiatrist isn't in the nurse's scope of practice. Social workers help clients with financial concerns; the scenario doesn't indicate that the client has a financial concern warranting a social worker at this time.

A diabetes nurse educator is teaching a group of clients with type 1 diabetes about "sick day rules." What guideline applies to periods of illness in a diabetic client?

Do not eliminate insulin when nauseated and vomiting.

Which of the following would be included in the teaching plan for a patient diagnosed with diabetes mellitus?

Elevated blood glucose levels contribute to complications of diabetes, such as diminished vision. Explanation: When blood glucose levels are well controlled, the potential for complications of diabetes is reduced. Several types of foods contain sugar, including cereals, sauces, salad dressing, fruit, and fruit juices. It is not feasible, nor is it advisable, to remove all sources of sugar from the diet. If the diabetes had been well controlled without insulin prior to the period of acute stress causing the need for insulin, the patient may be able to resume previous methods for control of diabetes when the stress is resolved.

After teaching a group of students about diabetic ketoacidosis (DKA) and hyperosmolar hyperglycemic nonketotic syndrome (HHNKS), the instructor determines that additional teaching is needed when the students identify which of the following as characteristic of HHNKS?

Elevated serum potassium levels Explanation: With HHNKS, blood glucose are significantly increased, well over 500 mg/dL, blood pH remains within the normal range of 7.35 to 7.45, and serum potassium and sodium levels are low.

When caring for the patient with acute pancreatitis, the nurse must consider pain relief measures. What nursing interventions could the nurse provide? (Select all that apply.)

Encouraging bed rest to decrease the metabolic rate Withholding oral feedings to limit the release of secretin Administering parenteral opioid analgesics as ordered

Which of the following would the nurse most likely assess in a client with diabetes who is experiencing autonomic neuropathy?

Erectile dysfunction Explanation: Autonomic neuropathy affects organ functioning. According the American Diabetes Association, up to 50% of men with diabetes develop erectile dysfunction when nerves that promote erection become impaired. Skeletal deformities and soft tissue ulcers may occur with motor neuropathy. Paresthesias are associated with sensory neuropathy.

A nurse is teaching a client with type 1 diabetes how to treat adverse reactions to insulin. To reverse hypoglycemia, the client ideally should ingest an oral carbohydrate. However, this treatment isn't always possible or safe. Therefore, the nurse should advise the client to keep which alternate treatment on hand? a) Glucagon b) Hydrocortisone c) 50% dextrose d) Epinephrine

Glucagon During a hypoglycemic reaction, a layperson may administer glucagon, an antihypoglycemic agent, to raise the blood glucose level quickly in a client who can't ingest an oral carbohydrate. Epinephrine isn't a treatment for hypoglycemia. Although 50% dextrose is used to treat hypoglycemia, it must be administered I.V. by a skilled health care professional. Hydrocortisone takes a relatively long time to raise the blood glucose level and therefore isn't effective in reversing hypoglycemia.

During a follow-up visit 3 months after a new diagnosis of type 2 diabetes, a client reports exercising and following a reduced-calorie diet. Assessment reveals that the client has only lost 1 pound and did not bring the glucose-monitoring record. Which value should the nurse measure? A. Glycosylated hemoglobin level B. Fasting blood glucose level C. Glucose via a urine dipstick test D. Glucose via an oral glucose tolerance test

Glycosylated hemoglobin level Glycosylated hemoglobin is a blood test that reflects the average blood glucose concentration over a period of approximately 2 to 3 months. When blood glucose is elevated, glucose molecules attach to hemoglobin in red blood cells. The longer the amount of glucose in the blood remains above normal, the more glucose binds to hemoglobin and the higher the glycosylated hemoglobin level becomes.

A client has just been diagnosed with hepatitis A. On assessment, the nurse expects to note:

Hallmark signs and symptoms of hepatitis A include anorexia, nausea, vomiting, fatigue, and weakness

A nurse is preparing a presentation for a local community group about hepatitis. Which of the following would the nurse include?

Hepatitis C increases a person's risk for liver cancer. Explanation: Infection with hepatitis C increases the risk of a person developing hepatic (liver) cancer. Hepatitis A is transmitted primarily by the oral-fecal route; hepatitis B is frequently spread by sexual contact and infected blood. Hepatitis E is similar to hepatitis A whereas hepatitis G is similar to hepatitis C.

A client has been brought to the ED by paramedics after bing found unconscious. The clients medic alert bracelet indicates that the client has type 1 diabetes and the clients blood glucose is 22 mg/dl. The use should anticipate what intervention?

IV administration of 50% dextrose in water

The nurse is educating the client with diabetes on setting up a sick plan to manage blood glucose control during times of minor illness such as influenza. Which is the most important teaching item to include? Take half the usual dose of insulin until symptoms resolve. Decrease food intake until nausea passes. Do not take insulin if not eating. Increase frequency of glucose self-monitoring.

Increase frequency of glucose self-monitoring. Minor illnesses such as influenza can present a special challenge to a diabetic client. The body's need for insulin increases during illness. Therefore, the client should take the prescribed insulin dose, increase the frequency of glucose monitoring, and maintain adequate fluid intake to counteract the dehydrating effects of hyperglycemia. Clear liquids and juices are encouraged. Taking less than normal dose of insulin may lead to ketoacidosis.

A nurse is providing education to a client who is newly diagnosed with diabetes mellitus. What are classic symptoms associated with diabetes? a) Increased thirst, hunger, and urination b) Loss of appetite, increased urination, and dehydration c) Increased weight loss, dehydration, and fatigue d) Increased weight gain, appetite, and thirst

Increased thirst, hunger, and urination

For a client with hyperglycemia, which assessment finding best supports a nursing diagnosis of Deficient fluid volume?

Increased urine osmolarity Explanation: In hyperglycemia, urine osmolarity (the measurement of dissolved particles in the urine) increases as glucose particles move into the urine. The client experiences glucosuria and polyuria, losing body fluids and experiencing deficient fluid volume. Cool, clammy skin; jugular vein distention; and a decreased serum sodium level are signs of fluid volume excess, the opposite imbalance.

A nurse is assessing a client who is receiving total parenteral nutrition (TPN). Which finding suggests that the client has developed hyperglycemia? A. Cheyne-Stokes respirations B. Diaphoresis C. Decreased appetite D. Increased urine output

Increased urine output Glucose supplies most of the calories in TPN; if the glucose infusion rate exceeds the client's rate of glucose metabolism, hyperglycemia arises. When the renal threshold for glucose reabsorption is exceeded, osmotic diuresis occurs, causing an increased urine output. A decreased appetite and diaphoresis suggest hypoglycemia, not hyperglycemia. Cheyne-Stokes respirations are characterized by a period of apnea lasting 10 to 60 seconds, followed by gradually increasing depth and frequency of respirations. Cheyne-Stokes respirations typically occur with cerebral depression or heart failure.

The client who is managing diabetes through diet and insulin control asks the nurse why exercise is important. Which is the best response by the nurse to support adding exercise to the daily routine? A. Decreases need for pancreas to produce more cells B. Increases ability for glucose to get into the cell and lowers blood sugar C. Creates an overall feeling of well-being and lowers risk of depression D. Decreases risk of developing insulin resistance and hyperglycemia

Increases ability for glucose to get into the cell and lowers blood sugar Exercise increases trans membrane glucose transporter levels in the skeletal muscles. This allows the glucose to leave the blood and enter into the cells where it can be used as fuel. Exercise can provide an overall feeling of well-being but is not the primary purpose of including in the daily routine of diabetic clients. Exercise does not stimulate the pancreas to produce more cells. Exercise can promote weight loss and decrease risk of insulin resistance but not the primary reason for adding to daily routine.

A nurse explains to a client that she will administer his first insulin dose in his abdomen. How does absorption at the abdominal site compare with absorption at other sites? a) Insulin is absorbed rapidly regardless of the injection site. b) Insulin is absorbed more rapidly at abdominal injection sites than at other sites. c) Insulin is absorbed unpredictably at all injection sites. d) Insulin is absorbed more slowly at abdominal injection sites than at other sites.

Insulin is absorbed more rapidly at abdominal injection sites than at other sites. Subcutaneous insulin is absorbed most rapidly at abdominal injection sites, more slowly at sites on the arms, and slowest at sites on the anterior thigh. Absorption after injection in the buttocks is less predictable.

Which factor presents the most likely cause for weight gain in a diabetic client who is controlled with insulin?

Insulin is an anabolic hormone. Insulin is an anabolic hormone that is known to cause weight gain

NPH is an example of which type of insulin? Short-acting Rapid-acting Intermediate-acting Long-acting

Intermediate-acting NPH is an intermediate-acting insulin.

Which of the following statements is correct regarding glargine (Lantus) insulin?

It cannot be mixed with any other type of insulin. Explanation: Because this insulin is in a suspension with a pH of 4, it cannot be mixed with other insulins because this would cause precipitation. There is not a peak in action. It is approved to give once daily

The nurse is describing the action of insulin in the body to a client newly diagnosed with type 1 diabetes. Which of the following would the nurse explain as being the primary action? A. It stimulates the pancreatic beta cells. B. It decreases the intestinal absorption of glucose. C. It aids in the process of gluconeogenesis. D. It carries glucose into body cells.

It carries glucose into body cells Insulin carries glucose into body cells as their preferred source of energy. Besides, it promotes the liver's storage of glucose as glycogen and inhibits the breakdown of glycogen back into glucose. Insulin does not aid in gluconeogenesis but inhibits the breakdown of glycogen back into glucose. Insulin does not have an effect on the intestinal absorption of glucose.

A client with a history of type 1 diabetes is demonstrating fast, deep, labored breathing and has fruity odored breath. What could be the cause of the client's current serious condition?

Ketoacidosis Kussmaul respirations (fast, deep, labored breathing) are common in ketoacidosis. Acetone, which is volatile, can be detected on the breath by its characteristic fruity odor. If treatment is not initiated, the outcome of ketoacidosis is circulatory collapse, renal shutdown, and death. Ketoacidosis is more common in people with diabetes who no longer produce insulin, such as those with type 1 diabetes. The most likely cause is ketoacidosis. People with type 2 diabetes are more likely to develop hyperosmolar hyperglycemic nonketotic syndrome because with limited insulin, they can use enough glucose to prevent ketosis but not enough to maintain a normal blood glucose level. The most likely cause is ketoacidosis

Which type of insulin acts most quickly? Glargine NPH Lispro Regular

Lispro Explanation: The onset of action of rapid-acting lispro is within 10 to 15 minutes. The onset of action of short-acting regular insulin is 30 minutes to 1 hour. The onset of action of intermediate-acting NPH insulin is 3 to 4 hours. The onset of action of very long-acting glargine is ~6 hours.

A client is being prepared to undergo laboratory and diagnostic testing to confirm the diagnosis of cirrhosis. Which test would the nurse expect to be used to provide definitive confirmation of the disorder?

Liver biopsy

A client with diabetes mellitus is prescribed to switch from animal to synthesized human insulin. Which of the following factors should the nurse monitor when caring for the client?

Low blood glucose levels Explanation: Clients who switch from animal to synthesized human insulin should be monitored for low blood glucose levels initially because the human form of insulin is used more effectively.

A patient with a diagnosis of type 2 diabetes has been vigilant about glycemic control since being diagnosed and has committed to increasing her knowledge about the disease. To reduce her risk of developing diabetic nephropathy in the future, this patient should combine glycemic control with what other preventative measure? a) Maintenance of a low-sodium, low-protein diet b) Vigorous physical activity at least three times weekly c) Maintenance of healthy blood pressure and prompt treatment of hypertension d) Subcutaneous injection of 5,000 units of heparin twice daily

Maintenance of healthy blood pressure and prompt treatment of hypertension

The nurse is educating the patient with diabetes about the importance of increasing dietary fiber. What should the nurse explain is the rationale for the increase? (select all that apply) a. may improve blood glucose levels b. decrease the need for exogenous insulin c. help reduce cholesterol levels d. may reduce postprandial glucose levels e. increase potassium levels

May improve blood glucose levels Decrease the need for exogenous insulin Help reduce cholesterol levels

A client with diabetes is receiving an oral anti diabetic agent that acts to help the tissues use available insulin more efficiently. Which of the following agents would the nurse expect to administer? A. Glipizide B. Glyburide C. Repaglinide D. Metformin

Metformin Metformin is a biguanide and, along with the thiazolidinediones (rosiglitazone and pioglitazone), are categorized as insulin sensitizers; they help tissues use available insulin more efficiently. Glyburide and glipizide, which are sulfonylureas, and repaglinide, a meglitinide, are described as being insulin releasers because they stimulate the pancreas to secrete more insulin.

A group of students are reviewing the various types of drugs that are used to treat diabetes mellitus. The students demonstrate understanding of the material when they identify which of the following as an example of an alpha-glucosidase inhibitor?

Miglitol Explanation: Alpha-glucosidase inhibitors include drugs such as miglitol and acarbose. Metformin is a biguanide. Glyburide is a sulfonylurea. Rosiglitazone is a thiazolidinedione.

A client with type 1 diabetes is experiencing polyphagia. The nurse knows to assess for which additional clinical manifestation(s) associated with this classic symptom? Weight gain Dehydration Altered mental state Muscle wasting and tissue loss

Muscle wasting and tissue loss Polyphagia results from the catabolic state induced by insulin deficiency and the breakdown of proteins and fats. Although clients with type 1 diabetes may experience polyphagia (increased hunger), they may also exhibit muscle wasting, subcutaneous tissue loss, and weight loss due to impaired glucose and protein metabolism and impaired fatty acid storage.

A nurse expects to find which signs and symptoms in a client experiencing hypoglycemia? A. Polyphagia and flushed, dry skin B. Polyuria, headache, and fatigue C. Polydipsia, pallor, and irritability D. Nervousness, diaphoresis, and confusion

Nervousness, diaphoresis, and confusion Signs and symptoms associated with hypoglycemia include nervousness, diaphoresis, weakness, light-headedness, confusion, paresthesia, irritability, headache, hunger, tachycardia, and changes in speech, hearing, or vision. If untreated, signs and symptoms may progress to unconsciousness, seizures, coma, and death. Polydipsia, polyuria, and polyphagia are symptoms associated with hyperglycemia.

A characteristic of type 2 diabetes includes which of the following? A. Little insulin B. No islet cell antibodies C. Ketosis-prone when insulin absent D. Often have islet antibodies

No Islet cell antibodies Type 2 diabetes is characterized by no islet cell antibodies or a decrease in endogenous insulin or increase with insulin resistance. Type 1 diabetes is characterized by production of little or no insulin; the patient is ketosis-prone when insulin is absent and often has islet cell antibodies.

A nurse obtains a fingerstick glucose level of 45 mg/dl on a client newly diagnosed with diabetes mellitus. The client is alert and oriented, and the client's skin is warm and dry. How should the nurse intervene? A. Obtain a serum glucose level. B. Obtain a repeat fingerstick glucose level. C. Notify the physician. D. Give the client 4 oz of milk and a graham cracker with peanut butter.

Obtain a repeat fingerstick glucose level The nurse should recheck the fingerstick glucose level to verify the original result because the client isn't exhibiting signs of hypoglycemia. The nurse should give the client milk and a graham cracker with peanut butter or a glass of orange juice after confirming the low glucose level. It isn't necessary to notify the physician or to obtain a serum glucose level at this time.

Which of the following factors should the nurse take into consideration when planning meals and selecting the type and dosage of insulin or oral hypoglycemic agent for an elderly patient with diabetes mellitus?

Patient's eating and sleeping habits Explanation: The eating and sleeping habits of older adults differ from those of young or middle-aged persons. The nurse should take this into consideration when planning meals and selecting the proper type and dosage of insulin or oral hypoglycemic agent. The nurse should evaluate the patient's ability to self-administer insulin before developing a teaching program. Cognitive problems and patient history may not be taken into consideration when planning meals and selecting the proper type and dosage of insulin or oral hypoglycemic agent.

Which of the following clinical characteristics is associated with Type 1 diabetes (previously referred to as insulin-dependent diabetes mellitus [IDDM])?

Presence of islet cell antibodies Explanation: Individuals with type 1 diabetes often have islet cell antibodies. Individuals with type 1 diabetes are usually thin or demonstrate recent weight loss at the time of diagnosis. Individuals with type 1 diabetes are ketosis-prone when insulin is absent. Individuals with type 1 diabetes need insulin to preserve life.

Which clinical characteristic is associated with type 1 diabetes (previously referred to as insulin-dependent diabetes mellitus)? a) Requirement for oral hypoglycemic agents b) Presence of islet cell antibodies c) Obesity d) Rare ketosis

Presence of islet cell antibodies Explanation: Individuals with type 1 diabetes often have islet cell antibodies and are usually thin or demonstrate recent weight loss at the time of diagnosis. These individuals are prone to experiencing ketosis when insulin is absent and require exogenous insulin to preserve life.

The nurse is administering an insulin drip to a patient in ketoacidosis. What insulin does the nurse know is the only one that can be used intravenously? A. Regular B. Lantus C. Lispro D. NPH

Regular Short-acting insulins are called regular insulin (marked R on the bottle). Regular insulin is a clear solution and is usually administered 20 to 30 minutes before a meal, either alone or in combination with a longer-acting insulin. Regular insulin is the only insulin approved for IV use.

A patient has been newly diagnosed with type 2 diabetes, and the nurse is assisting with the development of a meal plan. What step should be taken into consideration prior to making the meal plan? a) Determining whether the patient is on insulin or taking oral antidiabetic medication b) Ensuring that the patient understands that some favorite foods may not be allowed on the meal plan and substitutes will need to be found c) Reviewing the patient's diet history to identify eating habits and lifestyle and cultural eating patterns d) Making sure that the patient is aware that quantity of foods will be limited

Reviewing the patient's diet history to identify eating habits and lifestyle and cultural eating patterns The first step in preparing a meal plan is a thorough review of the patient's diet history to identify eating habits and lifestyle and cultural eating patterns.

The nurse is preparing a care plan for a client with hepatic cirrhosis. Which nursing diagnoses are appropriate? Select all that apply.

Risk for injury related to altered clotting mechanisms Activity intolerance related to fatigue, general debility, muscle wasting, and discomfort Disturbed body image related to changes in appearance, sexual dysfunction, and role function

The diabetic client asks the nurse why shoes and socks are removed at each office visit. Which assessment finding is most significant in determining the protocol for inspection of feet? A. Sensory neuropathy B. Retinopathy C. Autonomic neuropathy D. Nephropathy

Sensory neuropathy Neuropathy results from poor glucose control and decreased circulation to nerve tissues. Neuropathy involving sensory nerves located in the periphery can lead to lack of sensitivity, which increases the potential for soft tissue injury without client awareness. The feet are inspected on each visit to insure no injury or pressure has occurred. Autonomic neuropathy, retinopathy, and nephropathy affect nerves to organs other than feet.

A client with diabetes mellitus has a prescription for 5 units of U-100 regular insulin and 25 units of U-100 isophane insulin suspension (NPH) to be taken before breakfast. At about 4:30 p.m., the client experiences headache, sweating, tremor, pallor, and nervousness. What is the most probable cause of these signs and symptoms? A. Serum glucose level of 52 mg/dl B. Serum calcium level of 10.2 mg/dl C. Serum glucose level of 450 mg/dl D. Serum calcium level of 8.9 mg/dl

Serum glucose level of 52 mg/dL Headache, sweating, tremor, pallor, and nervousness typically result from hypoglycemia, an insulin reaction in which serum glucose level drops below 70 mg/dl. Hypoglycemia may occur 4 to 18 hours after administration of isophane insulin suspension or insulin zinc suspension (Lente), which are intermediate-acting insulins. Although hypoglycemia may occur at any time, it usually precedes meals. Hyperglycemia, in which serum glucose level is above 180 mg/dl, causes such early manifestations as fatigue, malaise, drowsiness, polyuria, and polydipsia. A serum calcium level of 8.9 mg/dl or 10.2 mg/dl is within normal range and wouldn't cause the client's symptoms.

Laboratory studies indicate a client's blood glucose level is 185 mg/dl. Two hours have passed since the client ate breakfast. Which test would yield the most conclusive diagnostic information about the client's glucose use?

Serum glycosylated hemoglobin (Hb A1c)

A client with a tentative diagnosis of hyperosmolar hyperglycemic nonketotic syndrome (HHNS) has a history of type 2 diabetes that is being controlled with an oral diabetic agent, tolazamide (Tolinase). Which laboratory test is the most important for confirming this disorder?

Serum osmolarity Explanation: Serum osmolarity is the most important test for confirming HHNS; it's also used to guide treatment strategies and determine evaluation criteria. A client with HHNS typically has a serum osmolarity of more than 350 mOsm/L. Serum potassium, serum sodium, and ABG values are also measured, but they aren't as important as serum osmolarity for confirming a diagnosis of HHNS. A client with HHNS typically has hypernatremia and osmotic diuresis. ABG values reveal acidosis, and the potassium level is variable.

A client with type 2 diabetes normally achieves adequate glycemic control through diet and exercise. Upon being admitted to the hospital for a cholecystectomy, however, the client has required insulin injections on two occasions. The nurse would identify what likely cause for this short-term change in treatment?

Stress has likely caused an increase in the client's blood sugar levels.

Which of the following categories of oral antidiabetic agents exert their primary action by directly stimulating the pancreas to secrete insulin?

Sulfonylureas Explanation: A functioning pancreas is necessary for sulfonylureas to be effective. Thiazolidinediones enhance insulin action at the receptor site without increasing insulin secretion from the beta cells of the pancreas. Biguanides facilitate insulin's action on peripheral receptor sites. Alpha glucosidase inhibitors delay the absorption of glucose in the intestinal system, resulting in a lower postprandial blood glucose level.

Insulin is a hormone secreted by the Islets of Langerhans and is essential for the metabolism of carbohydrates, fats, and protein. The nurse understands the physiologic importance of gluconeogenesis, which refers to the:

Synthesis of glucose from noncarbohydrate sources.

Which of the following factors would a nurse identify as a most likely cause of diabetic ketoacidosis (DKA) in a client with diabetes? a) The client has eaten and has not taken or received insulin. b) The client has not consumed sufficient calories. c) The client continues medication therapy despite adequate food intake. d) The client has been exercising more than usual.

The client has eaten and has not taken or received insulin. If the client has eaten and has not taken or received insulin, DKA is more likely to develop. Hypoglycemia is more likely to develop if the client has not consumed food and continues to take insulin or oral antidiabetic medications, if the client has not consumed sufficient calories, or if client has been exercising more than usual.

Which may be a potential cause of hypoglycemia in the client diagnosed with diabetes mellitus? A. The client has eaten but has not taken or received insulin. B. The client has not eaten but continues to take insulin or oral antidiabetic medications. C. The client has not been exercising. D. The client has not complied with the prescribed treatment regimen.

The client has not eaten but continues to take insulin or oral antidiabetic medications. Hypoglycemia occurs when a client with diabetes is not eating and continues to take insulin or oral antidiabetic medications. Hypoglycemia does not occur when the client has not been compliant with the prescribed treatment regimen. If the client has eaten and has not taken or received insulin, diabetic ketoacidosis is more likely to develop.

A male client, aged 42, is diagnosed with diabetes mellitus. He visits the gym regularly and is a vegetarian. Which of the following factors is important when assessing the client? A. History of radiographic contrast studies that used iodine B. The client's exercise routine C. The client's mental and emotional status D. The client's consumption of carbohydrates

The client's consumption of carbohydrates While assessing a client, it is important to note the client's consumption of carbohydrates because he has high blood sugar. Although other factors such as the client's mental and emotional status, history of tests involving iodine, and exercise routine can be part of data collection, they are not as important to information related to the client's to be noted in a client with high blood sugar.

A pregnant women has been diagnosed with gestational diabetes. The client is shocked. The nurse explains that gestational diabetes is a result of what etiology factor?

The effects of hormonal changes during pregnancy

The pancreas continues to release a small amount of basal insulin overnight, while a person is sleeping. The nurse knows that if the body needs more sugar: The process of gluconeogenesis will be inhibited. Insulin will be released to facilitate the transport of sugar. The pancreatic hormone glucagon will stimulate the liver to release stored glucose. Glycogenesis will be decreased by the liver.

The pancreatic hormone glucagon will stimulate the liver to release stored glucose. When sugar levels are low, glucagon promotes hyperglycemia by stimulating the release of stored glucose. Glycogenolysis and gluconeogenesis will both be increased. Insulin secretion would promote hypoglycemia.

The greatest percentage of people have which type of diabetes? Type 2 Impaired glucose tolerance Type 1 Gestational

Type 2 Type 2 diabetes accounts for 90% to 95% of all diabetes. Type 1 accounts for 5% to 10% of all diabetes. Gestational diabetes has an onset during pregnancy. Impaired glucose tolerance is defined as an oral glucose tolerance test value between 140 mg/dL and 200 mg/dL.

Which of the following insulins has the longest onset of action?

Ultralente

A client with type 2 diabetes asks the nurse why he can't have a pancreatic transplant. Which of the following would the nurse include as a possible reason? A. Need for exocrine enzymatic drainage B. Need for lifelong immunosuppressive therapy C. Underlying problem of insulin resistance D. Increased risk for urologic complications

Underlying problem of insulin resistance Clients with type 2 diabetes are not offered the option of a pancreas transplant because their problem is insulin resistance, which does not improve with a transplant. Urologic complications or the need for exocrine enzymatic drainage are not reasons for not offering pancreas transplant to clients with type 2 diabetes. Any transplant requires lifelong immunosuppressive drug therapy and is not the factor.

A controlled type 2 diabetic client states, "The doctor said if my blood sugars remain stable, I may not need to take any medication." Which response by the nurse is most appropriate? a) "Diet, exercise, and weight loss can eliminate the need for medication." b) "You will be placed on a strict low-sugar diet for better control." c) "You misunderstood the doctor. Let's ask for clarification." d) "Some doctors do not treat blood sugar elevation until symptoms appear."

a) "Diet, exercise, and weight loss can eliminate the need for medication."

A nurse is caring for a client with diabetes mellitus. The client has a blood glucose level of 40 mg/dL. Which of the following rapidly absorbed carbohydrate would be most effective? a) 1/2 cup fruit juice or regular soft drink b) 1/2 tbsp honey or syrup c) Three to six LifeSavers candies d) 4 oz of skim milk

a) 1/2 cup fruit juice or regular soft drink In a client with hypoglycemia, the nurse uses the rule of 15: Give 15 g of rapidly absorbed carbohydrate, wait 15 minutes, recheck the blood sugar, and administer another 15 g of glucose if the blood sugar is not above 70 mg/dL. One-half cup fruit juice or regular soft drink is equivalent to the recommended 15 g of rapidly absorbed carbohydrate. Eight ounces of skim milk is equivalent to the recommended 15 g of rapidly absorbed carbohydrate. One tablespoon of honey or syrup is equivalent to the recommended 15 g of rapidly absorbed carbohydrate. Six to eight LifeSavers candies is equivalent to the recommended 15 g of rapidly absorbed carbohydrate.

A patient is admitted with diabetic ketoacidosis (DKA). The physician writes all of the following orders. Which order should the nurse implement first? a) Infuse 0.9% normal saline solution 1 L/hr for 2 hours. b) Administer sodium bicarbonate 50 mEq IV push. c) Start an infusion of regular insulin at 50 U/hr. d) Administer regular insulin 30 U IV push.

a) Infuse 0.9% normal saline solution 1 L/hr for 2 hours. Explanation: In addition to treating hyperglycemia, management of DKA is aimed at correcting dehydration, electrolyte loss, and acidosis before correcting the hyperglycemia with insulin. In dehydrated patients, rehydration is important for maintaining tissue perfusion. Initially, 0.9% sodium chloride (normal saline) solution is administered at a rapid rate, usually 0.5 to 1 L/hr for 2 to 3 hours.

A 53-year-old client is brought to the ED, via squad, where you practice nursing. He is demonstrating fast, deep, labored breathing and has a fruity odor to his breath. He has a history of type 1 diabetes. What could be the cause of his current serious condition? a) Ketoacidosis b) Hyperosmolar hyperglycemic nonketotic syndrome c) All options are correct d) Hepatic disorder

a) Ketoacidosis Kussmaul respirations (fast, deep, labored breathing) are common in ketoacidosis. Acetone, which is volatile, can be detected on the breath by its characteristic fruity odor. If treatment is not initiated, the outcome of ketoacidosis is circulatory collapse, renal shutdown, and death. Ketoacidosis is more common in people with diabetes who no longer produce insulin, such as those with type 1 diabetes.

A patient is diagnosed with type 1 diabetes. What clinical characteristics does the nurse expect to see in this patient? (Select all that apply.) a) Little endogenous insulin b) Ketosis-prone c) Older than 65 years of age d) Younger than 30 years of age e) Obesity at diagnoses

a) Little endogenous insulin b) Ketosis-prone d) Younger than 30 years of age Explanation: Type I diabetes mellitus is associated with the following characteristics: onset any age, but usually young (<30 y); usually thin at diagnosis, recent weight loss; etiology includes genetic, immunologic, and environmental factors (e.g., virus); often have islet cell antibodies; often have antibodies to insulin even before insulin treatment; little or no endogenous insulin; need exogenous insulin to preserve life; and ketosis prone when insulin absent.

A nurse is teaching a diabetic support group about the causes of type 1 diabetes. The teaching is determined to be effective when the group is able to attribute which of the following factors as a cause of type 1 diabetes? a) Presence of autoantibodies against islet cells b) Obesity c) Rare ketosis d) Altered glucose metabolism

a) Presence of autoantibodies against islet cells There is evidence of an autoimmune response in type 1 diabetes. This is an abnormal response in which antibodies are directed against normal tissues of the body, responding to these tissues as if they were foreign. Autoantibodies against islet cells and against endogenous (internal) insulin have been detected in people at the time of diagnosis and even several years before the development of clinical signs of type 1 diabetes.

The nurse is teaching a patient about self-administration of insulin and mixing of regular and neutral protamine Hagedorn (NPH) insulin. Which of the following is important to include in the teaching plan? a) When mixing insulin, the regular insulin is drawn up into the syringe first. b) There is no longer a need to inject air into the bottle of insulin before insulin is withdrawn. c) If two different types of insulin are ordered, they need to be given in separate injections. d) When mixing insulin, the NPH insulin is drawn up into the syringe first.

a) When mixing insulin, the regular insulin is drawn up into the syringe first. Explanation: When rapid-acting or short-acting insulins are to be given simultaneously with longer-acting insulins, they are usually mixed together in the same syringe; the longer-acting insulins must be mixed thoroughly before drawing into the syringe. The American Diabetic Association (ADA) recommends that the regular insulin be drawn up first. The most important issues are (1) that patients are consistent in technique, so the wrong dose is not drawn in error or the wrong type of insulin, and (2) that patients not inject one type of insulin into the bottle containing a different type of insulin. Injecting cloudy insulin into a vial of clear insulin contaminates the entire vial of clear insulin and alters its action.

A nurse is assigned to care for a patient who is suspected of having type 2 diabetes. Select all the clinical manifestations that the nurse knows could be consistent with this diagnosis. a) Wounds that heal slowly or respond poorly to treatment b) Blurred or deteriorating vision c) Sudden weight loss and anorexia d) Fatigue and irritability e) Polyuria and polydipsia

a) Wounds that heal slowly or respond poorly to treatment b) Blurred or deteriorating vision d) Fatigue and irritability e) Polyuria and polydipsia

A 16 year old patient newly diagnosed with type 1 diabetes has a very low body weight despite eating regular meals. The patient is upset because friends frequently state, "You look anorexic." Which of the following statements would be the best response by the nurse to help this patient understand the cause of weight loss due to this condition? a."your body is using protein and fat for energy instead of glucose" b. "don't worry about what your friends think; the carbohydrates you eat are being quickly digested, increasing your metabolism" c. "I will refer you to a dietician who can help you with your weight" d. " you may be having undiagnosed infections causing you to lose extra weight"

a. "your body is using protein and fat for energy instead of glucose" Explanation: Persons with type 1 diabetes, particularly those in poor control of the condition, tend to be thin because when the body cannot effectively utilize glucose for energy (no insulin supply), it begins to break down protein and fat as an alternate energy source. Patients may be underweight at the onset of type 1 diabetes because of rapid weight loss from severe hyperglycemia. The goal initially may be to provide a higher-calorie diet to regain lost weight and blood glucose control.

A female diabetic patient who weighs 130b has an ideal body weight of 116 lb. For weight reduction of 2 lb/week, approximately what should her daily caloric intake be? a. 1000 calories b. 1200 calories c. 1500 calories d. 1800 calories

a. 1000 calories

What pharmacologic therapy does the nurse anticipate administering when the patient is experiencing thyroid storm? (Select all that apply.)

a. acetaminophen b. iodine c. propylthiouracil

The nurse is caring for a patient with an abnormally low blood glucose concentration. What glucose level will the nurse observe when assessing laboratory results? a. lower than 50-60 mg/dl b. between 60 and 80 mg/dL c. Between 75 and 90 mg/dL d. 95 mg/dL

a. lower than 50-60 mg/dl

During a class on exercise for clients with diabetes mellitus, a client asks the nurse educator how often to exercise. To meet the goals of planned exercise, the nurse educator should advise the client to exercise: A. at least three times per week. B. at least five times per week. C. every day. D. at least once per week.

at least three times per week Clients with diabetes must exercise at least three times per week to meet the goals of planned exercise — lowering the blood glucose level, reducing or maintaining the proper weight, increasing the serum high-density lipoprotein level, decreasing serum triglyceride levels, reducing blood pressure, and minimizing stress. Exercising once per week wouldn't achieve these goals. Exercising more than three times per week, although beneficial, would exceed the minimum requirement.

A patient with type 2 diabetes has recently been placed on acarbose (Precose); the nurse is explaining how to take this medication. The teaching is determined to be effective based on which of the following statements? a) "I will take this medication in the morning, 15 minutes before breakfast." b) "I will take this medication in the morning, with my first bite of breakfast." c) "This medication needs to be taken after the midday meal." d) "It does not matter what time of day I take this medication."

b) "I will take this medication in the morning, with my first bite of breakfast." Explanation: Alpha-glucosidase inhibitors, such as acarbose (Precose) and miglitol (Glyset), delay absorption of complex carbohydrates in the intestine and slow entry of glucose into systemic circulation. They must be taken with the first bite of food to be effective.

Which instruction should a nurse give to a client with diabetes mellitus when teaching about "sick day rules"? a) "Don't take your insulin or oral antidiabetic agent if you don't eat." b) "Test your blood glucose every 4 hours." c) "Follow your regular meal plan, even if you're nauseous." d) "It's okay for your blood glucose to go above 300 mg/dl while you're sick."

b) "Test your blood glucose every 4 hours." The nurse should instruct a client with diabetes mellitus to check his blood glucose levels every 3 to 4 hours and take insulin or an oral antidiabetic agent as usual, even when he's sick. If the client's blood glucose level rises above 300 mg/dl, he should call his physician immediately. If the client is unable to follow the regular meal plan because of nausea, he should substitute soft foods, such as gelatin, soup, and custard.

Which of the following should be included in the teaching plan for a patient receiving glargine (Lantus), "peakless" basal insulin? a) Administer the total daily dosage in two doses. b) Do not mix with other insulins. c) Draw up the drug first, then add regular insulin. d) It is rapidly absorbed, has a fast onset of action.

b) Do not mix with other insulins. Explanation: Because glargine is in a suspension with a pH of 4, it cannot be mixed with other insulins because this would cause precipitation. When administering glargine (Lantus) insulin it is very important to read the label carefully and to avoid mistaking Lantus insulin for Lente insulin and vice versa.

A nurse is teaching a patient recovering from diabetic ketoacidosis (DKA) about management of "sick days." The patient asks the nurse why it is important to monitor the urine for ketones. Which of the following statements is the nurse's best response? a) Ketones are formed when insufficient insulin leads to cellular starvation. As cells rupture, they release these acids into the blood. b) Ketones accumulate in the blood and urine when fat breaks down. Ketones signal a deficiency of insulin that will cause the body to start to break down stored fat for energy. c) Excess glucose in the blood is metabolized by the liver and turned into ketones, which are an acid. d) When the body does not have enough insulin hyperglycemia occurs. Excess glucose is broken down by the liver, causing acidic byproducts to be released.

b) Ketones accumulate in the blood and urine when fat breaks down. Ketones signal a deficiency of insulin that will cause the body to start to break down stored fat for energy Ketones (or ketone bodies) are byproducts of fat breakdown, and they accumulate in the blood and urine. Ketones in the urine signal a deficiency of insulin and control of type 1 diabetes is deteriorating. When almost no effective insulin is available, the body starts to break down stored fat for energy.

An older adult patient that has diabetes type 2 comes to the emergency department with second-degree burns to the bottom of both feet and states, "I didn't feel too hot but my feet must have been too close to the heater." What does the nurse understand is most likely the reason for the decrease in temperature sensation? a) Autonomic neuropathy b) Peripheral neuropathy c) A faulty heater d) Sudomotor neuropathy

b) Peripheral neuropathy

After taking glipizide (Glucotrol) for 9 months, a client experiences secondary failure. What should the nurse expect the physician to do? a) Restrict carbohydrate intake to less than 30% of the total caloric intake. b) Switch the client to a different oral antidiabetic agent. c) Initiate insulin therapy. d) Order an additional oral antidiabetic agent.

b) Switch the client to a different oral antidiabetic agent The nurse should anticipate that the physician will order a different oral antidiabetic agent. Many clients (25% to 60%) who take glipizide respond to a different oral antidiabetic agent. Therefore, it wouldn't be appropriate to initiate insulin therapy at this time. However, if a new oral antidiabetic agent is unsuccessful in keeping glucose levels at an acceptable level, insulin may be used in addition to the antidiabetic agent. Restricting carbohydrate intake isn't necessary.

A client with newly diagnosed type 2 diabetes is admitted to the metabolic unit. The primary goal for this admission is education. Which goal should the nurse incorporate into her teaching plan? a) Maintenance of blood glucose levels between 180 and 200 mg/dl b) An eye examination every 2 years until age 50 c) Weight reduction through diet and exercise d) Smoking reduction but not complete cessation

c) Weight reduction through diet and exercise

A nurse is explaining the action of insulin to a client with diabetes mellitus. During client teaching, the nurse reviews the process of insulin secretion in the body. The nurse is correct when she states that insulin is secreted from the: a) alpha cells of the pancreas. b) beta cells of the pancreas. c) adenohypophysis. d) parafollicular cells of the thyroid.

b) beta cells of the pancreas. The beta cells of the pancreas secrete insulin. The adenohypophysis, or anterior pituitary gland, secretes many hormones, such as growth hormone, prolactin, thyroid-stimulating hormone, corticotropin, follicle-stimulating hormone, and luteinizing hormone, but not insulin. The alpha cells of the pancreas secrete glucagon, which raises the blood glucose level. The parafollicular cells of the thyroid secrete the hormone calcitonin, which plays a role in calcium metabolism.

Which of the following statements is true regarding gestational diabetes? a. it occurs in most pregnancies b. a glucose challenge test should be performed between 24 and 28 weeks. c. Its onset is usually in the first trimester. d. there is a low risk for perinatal

b. a glucose challenge test should be performed between 24 and 28 weeks.

The nurse is assessing a patient with non proliferative (background) retinopathy. When examining the retina, what would the nurse expect to assess? (select all that apply) a. Detachment b. leakage of fluid or serum (exudates) c.Focal capillary single closure d. Blurred optic discs e. Microaneurysms

b. leakage of fluid or serum (exudates) c.Focal capillary single closure e. Microaneurysms

A client diagnosed with hyperosmolar hyperglycemic nonketotic syndrome (HHNS) is stabilized and prepared for discharge. When preparing the client for discharge and home management, which statement indicates that the client understands his condition and how to control it? a) "I will have to monitor my blood glucose level closely and notify the physician if it's constantly elevated." b) "If I experience trembling, weakness, and headache, I should drink a glass of soda that contains sugar." c) "I can avoid getting sick by not becoming dehydrated and by paying attention to my need to urinate, drink, or eat more than usual." d) "If I begin to feel especially hungry and thirsty, I'll eat a snack high in carbohydrates.

c) "I can avoid getting sick by not becoming dehydrated and by paying attention to my need to urinate, drink, or eat more than usual." The client stating that he'll remain hydrated and pay attention to his eating, drinking, and voiding needs indicates understanding of HHNS. Inadequate fluid intake during hyperglycemic episodes commonly leads to HHNS. By recognizing the signs of hyperglycemia (polyuria, polydipsia, and polyphagia) and increasing fluid intake, the client may prevent HHNS. Drinking a glass of nondiet soda would be appropriate for hypoglycemia. A client whose diabetes is controlled with oral antidiabetic agents usually doesn't need to monitor blood glucose levels. A high-carbohydrate diet would exacerbate the client's condition, particularly if fluid intake is low.

A nurse is teaching a client recovering from diabetic ketoacidosis (DKA) about management of "sick days." The client asks the nurse why it is important to monitor the urine for ketones. Which statement is the nurse's best response? a) "Ketones are formed when insufficient insulin leads to cellular starvation. As cells rupture, they release these acids into the blood." b) "Excess glucose in the blood is metabolized by the liver and turned into ketones, which are an acid." c) "Ketones accumulate in the blood and urine when fat breaks down in the absence of insulin. Ketones signal an insulin deficiency that will cause the body to start breaking down stored fat for energy." d) "When the body does not have enough insulin, hyperglycemia occurs. Excess glucose is broken down by the liver, causing acidic by-products to be released."

c) "Ketones accumulate in the blood and urine when fat breaks down in the absence of insulin. Ketones signal an insulin deficiency that will cause the body to start breaking down stored fat for energy." Ketones (or ketone bodies) are by-products of fat breakdown in the absence of insulin, and they accumulate in the blood and urine. Ketones in the urine signal an insulin deficiency and that control of type 1 diabetes is deteriorating. When almost no effective insulin is available, the body starts to break down stored fat for energy.

A health care provider prescribes short-acting insulin for a patient, instructing the patient to take the insulin 20 to 30 minutes before a meal. The nurse explains to the patient that Humulin-R, taken at 6:30 AM will reach peak effectiveness by: a) 2:30 PM. b) 12:30 PM. c) 8:30 AM. d) 10:30 AM.

c) 8:30 AM Short-acting insulin reaches its peak effectiveness 2 to 3 hours after administration. See Table 30-3 in the text.

Which of the following statements is true regarding gestational diabetes? a) It occurs in most pregnancies. b) Its onset is usually in the first trimester. c) A glucose challenge test should be performed between 24 and 28 weeks. d) There is a low risk for perinatal complications.

c) A glucose challenge test should be performed between 24 and 28 weeks. Explanation: A glucose challenge test should be performed between 24 and 48 weeks. It occurs in 2% to 5% of all pregnancies. Onset is usually in the second or third trimester. There is an above-normal risk for perinatal complications.

Matt Thompson, a 37-year-old farmer, has been diagnosed with pre diabetes. Following his visit with his primary care provider, you begin your client education session to discuss treatment strategies. What can be the consequences of untreated pre diabetes? a) Cardiac disease b) CVA c) All options are correct. d) Type 2 diabetes

c) All options are correct. The NIDDK has developed criteria that identify people with prediabetes, which can lead to type 2 diabetes, heart disease, and stroke.

A patient is admitted to the health care center with abdominal pain, nausea, and vomiting. The medical reports indicate a history of type 1 diabetes. The nurse suspects the patient's symptoms to be that of diabetic ketoacidosis (DKA). Which of the following actions will help the nurse confirm the diagnosis? a) Assessing the patient's ability to take a deep breath b) Assessing for excessive sweating c) Assessing the patient's breath odor d) Assessing the patient's ability to move all extremities

c) Assessing the patient's breath odor DKA is commonly preceded by a day or more of polyuria, polydipsia, nausea, vomiting, and fatigue with eventual stupor and coma if not treated. The breath has a characteristic fruity odor due to the presence of ketoacids. Checking the patient's breath will help the nurse confirm the diagnosis.

The nurse is discussing the new medication that a client will be taking for treatment of rheumatoid arthritis. Which disease-modifying antirheumatic drug (DMARD) will the nurse educate the client about? a) Infliximab (Remicade) b) Methylprednisolone (Medrol) c) Methotrexate (Rheumatrex) d) Etanercept (Enbrel)

c) Methotrexate (Rheumatrex) Methotrexate is a DMARD that reduces the amount of joint damage and slows the damage to other tissues as well. Etanercept and Infliximab are TNF-alpha inhibitors that reduce pain and inflammation. Methylprednisolone is a steroid to reduce pain and inflammation and slow joint destruction.

The nurse is preparing to administer IV fluids for a patient with ketoacidosis who has a history of hypertension and congestive heart failure. What order for fluids would the nurse anticipate infusing for this patient? a. D5W b. 0.9% normal saline c. 0.45 normal saline d. D5 normal saline

c. 0.45 normal saline

A patient with type 2 diabetes complains about waking up in the middle of the night nervous and confused, with tremors, sweating, and a feeling of hunger. Morning fasting blood sugar readings have been 110 to 140 m/dL; the patient admits to exercising excessively and skipping meals over the past several weeks. Based on these symptoms, the nurse will plan to instruct the patient to do which of the following? a. eat complex carbohydrate snack in the evening before bed. b. skip the evening NPH insulin dose on days when exercising and skipping meals. c. Check blood glucose at 3:00 in the morning d. Administer an increased dose of neutral protamine Hagedorn (NPH) insulin in the evening

c. Check blood glucose at 3:00 in the morning

A nurse is teaching a patient recovering from diabetic ketoacidosis (DKA) about management of "sick days". The patient asked the nurse why it is important to monitor the urine for ketones. Which of the following statements is the nurse's best response? a. when the body does not have enough insulin hyperglycemia occurs. Excess glucose is broken down by the liver, causing acidic byproducts to be released b. Ketones are formed when insufficient insulin leads to cellular starvation. As cells rupture, they release these acids into the blood. c. Ketones accumulate in the blood and urine when fat breaks down. Ketones signal a deficiency of insulin that will cause the body to start to break down stored fat for energy. d. Excess glucose in the blood is metabolized by the liver and turned into ketones, which are an acid

c. Ketones accumulate in the blood and urine when fat breaks down. Ketones signal a deficiency of insulin that will cause the body to start to break down stored fat for energy.

For a client with hyperglycemia, which assessment finding best supports a nursing diagnosis of Deficient fluid volume? A. Increased urine osmolarity B. Cool, clammy skin C. Decreased serum sodium level D. Jugular vein distention

cool, clammy skin In hyperglycemia, urine osmolarity (the measurement of dissolved particles in the urine) increases as glucose particles move into the urine. The client experiences glucosuria and polyuria, losing body fluids and experiencing deficient fluid volume. Cool, clammy skin; jugular vein distention; and a decreased serum sodium level are signs of fluid volume excess, the opposite imbalance.

A 1,200-calorie diet and exercise are prescribed for a patient with newly diagnosed type 2 diabetes. The nurse is teaching the patient about meal planning using exchange lists. The teaching is determined to be effective based on which of the following statements? a) "For dinner I ate 4-ounces of sliced roast beef on a bagel with lettuce, tomato, and onion, 1 ounce low-fat cheese, 1 tablespoon mayonnaise, 1 cup fresh strawberry shortcake, and unsweetened iced tea." b) "For dinner I ate 2 ounces of sliced turkey, 1 cup mashed sweet potatoes, half a cup of carrots, half a cup of peas, a 3-ounce dinner roll, 1 medium banana, and a diet soda." c) "For dinner I ate 2 cups of cooked pasta with 3-ounces of boiled shrimp, 1 cup plum tomatoes, half a cup of peas and garlic-wine sauce, 2 cups fresh strawberries, and ice water with lemon." d) "For dinner I ate a 3-ounce hamburger on a bun, with ketchup, pickle, and onion, a green salad with 1 teaspoon Italian dressing, 1 cup of watermelon, and a diet soda."

d) "For dinner I ate a 3-ounce hamburger on a bun, with ketchup, pickle, and onion, a green salad with 1 teaspoon Italian dressing, 1 cup of watermelon, and a diet soda." Explanation: There are six main exchange lists: bread/starch, vegetable, milk, meat, fruit, and fat. Foods within one group (in the portion amounts specified) contain equal numbers of calories and are approximately equal in grams of protein, fat, and carbohydrate. Meal plans can be based on a recommended number of choices from each exchange list. Foods on one list may be interchanged with one another, allowing for variety while maintaining as much consistency as possible in the nutrient content of foods eaten. Example: 2 starch = 2 slices bread or a hamburger bun, 3 meat = 3 oz lean beef patty, 1 vegetable = green salad, 1 fat = 1 tbsp salad dressing, 1 fruit = 1¼ cup watermelon; "free" items like diet soda are optional.

A patient newly diagnosed with type 1 diabetes has an unusual increase in blood glucose from bedtime to morning. The physician suspects the patient is experiencing insulin waning. Based on this diagnosis, the nurse will expect which of the following changes to the patient's medication regimen? a) Changing the time of injection of evening intermediate-acting insulin from dinnertime to bedtime b) Decreasing evening bedtime dose of intermediate-acting insulin and administering a bedtime snack c) Increasing morning dose of long-acting insulin d) Administering a dose of intermediate-acting insulin before the evening meal

d) Administering a dose of intermediate-acting insulin before the evening meal Explanation: Insulin waning is a progressive rise in blood glucose form bedtime to morning. Treatment includes increasing the evening (predinner or bedtime) dose of intermediate-acting or long-acting insulin or instituting a dose of insulin before the evening meal if that is not already part of the treatment regimen.

A client is admitted with hyperosmolar hyperglycemic nonketotic syndrome (HHNS). Which laboratory finding should the nurse expect in this client? a) Arterial pH 7.25 b) Blood urea nitrogen (BUN) 15 mg/dl c) Plasma bicarbonate 12 mEq/L d) Blood glucose level 1,100 mg/dl

d) Blood glucose level 1,100 mg/dl HHNS occurs most frequently in older clients. It can occur in clients with either type 1 or type 2 diabetes mellitus but occurs most commonly in those with type 2. The blood glucose level rises to above 600 mg/dl in response to illness or infection. As the blood glucose level rises, the body attempts to rid itself of the excess glucose by producing urine. Initially, the client produces large quantities of urine. If fluid intake isn't increased at this time, the client becomes dehydrated, causing BUN levels to rise. Arterial pH and plasma bicarbonate levels typically remain within normal limits

Which of the following clinical manifestations of type 2 diabetes occurs if glucose levels are very high? a) Hyperactivity b) Oliguria c) Increased energy d) Blurred vision

d) Blurred vision Explanation: Blurred vision occurs when the blood glucose levels are very high. The other clinical manifestations are not consistent with type 2 diabetes.

A patient with type 1 diabetes complains about waking up in the middle of the night nervous and confused, with tremors, sweating, and a feeling of hunger. Morning fasting blood sugar readings have been 110 to 140 mg/dL; the patient admits to exercising excessively and skipping meals over the past several weeks. Based on these symptoms, the nurse will plan to instruct the patient to do which of the following? a) Administer an increased dose of neutral protamine Hagedorn (NPH) insulin in the evening. b) Eat a complex carbohydrate snack in the evening before bed. c) Skip the evening NPH insulin dose on days when exercising and skipping meals. d) Check blood glucose at 3:00 in the morning.

d) Check blood glucose at 3:00 in the morning. Explanation: In the Somogyi effect, the patient has normal or elevated blood glucose at bedtime, a decrease at 2 to 3 am to hypoglycemic levels, and a subsequent increase caused by the production of counterregulatory hormones. It is important to check the blood glucose in the early morning hours to detect the initial hypoglycemia.

Which of the following should be included in the teaching plan for a patient receiving glargine (Lantus),"peakless" basal insulin? a) It is rapidly absorbed, has a fast onset of action. b) Draw up the drug first, then add regular insulin. c) Administer the total daily dosage in two doses. d) Do not mix with other insulins.

d) Do not mix with other insulin Because glargine is in a suspension with a pH of 4, it cannot be mixed with other insulins because this would cause precipitation. When administering glargine (Lantus) insulin it is very important to read the label carefully and to avoid mistaking Lantus insulin for Lente insulin and vice versa.

Which of the following would be included in the teaching plan for a patient diagnosed with diabetes mellitus? a) The only diet change needed in the treatment of diabetes is to stop eating sugar. b) Sugar is found only in dessert foods. c) Once insulin injections are started in the treatment of type 2 diabetes, they can never be discontinued. d) Elevated blood glucose levels contribute to complications of diabetes, such as diminished vision.

d) Elevated blood glucose levels contribute to complications of diabetes, such as diminished vision.

A nurse is preparing to discharge a patient with coronary artery disease (CAD) and hypertension (HTN) who is at risk for type 2 diabetes. Which of the following information is important to include in the discharge teaching? a) How to recognize signs of diabetic ketoacidosis (DKA) b) How to self-inject insulin c) How to monitor ketones daily d) How to control blood glucose through lifestyle modification with diet and exercise

d) How to control blood glucose through lifestyle modification with diet and exercise Explanation: Persons at high risk for type 2 diabetes receive standard lifestyle recommendations plus metformin, standard lifestyle recommendations plus placebo, or an intensive program of lifestyle modifications. The 16-lesson curriculum of the intensive program of lifestyle modifications focuses on weight reduction of greater than 7% of initial body weight and physical activity of moderate intensity. It also includes behavior modification strategies designed to help patients achieve the goals of weight reduction and participation in exercise. These findings demonstrate that type 2 diabetes can be prevented or delayed in persons at high risk for the disease.

A patient with diabetes mellitus is receiving an oral antidiabetic agent. The nurse observes for which of the following symptoms when caring for this patient? a) Polydipsia b) Blurred vision c) Polyuria d) Hypoglycemia

d) Hypoglycemia Explanation: The nurse should observe the patient receiving an oral antidiabetic agent for the signs of hypoglycemia. The time when the reaction might occur is not predictable and could be from 30 to 60 minutes to several hours after the drug is ingested.

A 60-year-old patient comes to the ED with complaints of weakness, vision problems, increased thirst, increased urination, and frequent infections that do not seem to heal easily. The physician suspects that the patient has diabetes. Which of the following classic symptoms should the nurse watch for to confirm the diagnosis of diabetes? a) Numbness b) Fatigue c) Dizziness d) Increased hunger

d) Increased hunger Explanation: The classic symptoms of diabetes are the three Ps: polyuria (increased urination), polydipsia (increased thirst), and polyphagia (increased hunger). Some of the other symptoms include tingling, numbness, and loss of sensation in the extremities and fatigue.

A patient with type 1 diabetes is experiencing polyphagia. The nurse knows to assess for which additional clinical manifestations associated with this classic symptom? a) Altered mental state b) Weight gain c) Dehydration d) Muscle wasting and tissue loss

d) Muscle wasting and tissue loss Explanation: Polyphagia results from the catabolic state induced by insulin deficiency and the breakdown of proteins and fats. Although people with type 1 diabetes may experience polyphagia (increased hunger), they may also exhibit muscle wasting, subcutaneous tissue loss, and weight loss due to impaired glucose and protein metabolism and impaired fatty acid storage.

As a nurse educator, you have been invited to your local senior center to discuss health-maintaining strategies for older adults. During your education session on nutrition, you approach the subject of diabetes mellitus, its symptoms, and consequences. One of the women in your lecture group asks if you know the death rate from diabetes mellitus. What is your response? a) Ninth cause of death in the United States b) Tenth cause of death in the United States c) Fifth cause of death in the United States d) Seventh cause of death in the United States

d) Seventh cause of death in the United States

Which of the following may be a potential cause of hypoglycemia in the patient diagnosed with diabetes mellitus? a) The patient has not been compliant with the prescribed treatment regimen. b) The patient has consumed food and has not taken or received insulin. c) The patient has not been exercising. d) The patient has not consumed food and continues to take insulin or oral antidiabetic medications.

d) The patient has not consumed food and continues to take insulin or oral antidiabetic medications. Explanation: Hypoglycemia in patients is usually the result of too much insulin or delays in eating.

A client with diabetes mellitus develops sinusitis and otitis media accompanied by a temperature of 100.8° F (38.2° C). What effect do these findings have on his need for insulin? a) They decrease the need for insulin. b) They cause wide fluctuations in the need for insulin. c) They have no effect. d) They increase the need for insulin.

d) They increase the need for insulin. Insulin requirements increase in response to growth, pregnancy, increased food intake, stress, surgery, infection, illness, increased insulin antibodies, and some medications. Insulin requirements are decreased by hypothyroidism, decreased food intake, exercise, and some medications.

A client has type 1 diabetes. Her husband finds her unconscious at home and administers glucagon, 0.5 mg subcutaneously. She awakens in 5 minutes. Why should her husband offer her a complex carbohydrate snack as soon as possible? a) To stimulate her appetite b) To decrease the amount of glycogen in her system c) To decrease the possibility of nausea and vomiting d) To restore liver glycogen and prevent secondary hypoglycemia

d) To restore liver glycogen and prevent secondary hypoglycemia

Which of the following factors is the focus of nutrition intervention for patients with type 2 diabetes? a) Blood glucose level b) Protein metabolism c) Carbohydrate intake d) Weight loss

d) Weight loss Explanation: In most instances, people with type 2 diabetes require weight reduction; therefore, weight loss is the focus of nutrition intervention for patients with type 2 diabetes. A low-calorie diet may reduce clinical symptoms, and even a mild to moderate weight loss, such as 10 to 20 pounds, may lower blood glucose levels and improve insulin action.

Which of the following would be included in the teaching plan for a patient diagnosed with diabetes mellitus? a. once insulin injections are started for treatment of type 2 diabetes, they can never be discontinued b. sugar is found only in dessert foods c. The only diet change needed in the treatment of diabetes is to stop eating sugar d. elevated blood glucose levels contribute to complications of diabetes, such as diminished vision

d. elevated blood glucose levels contribute to complications of diabetes, such as diminished vision

A client tells the nurse that she has been working hard for the past 3 months to control her type 2 diabetes with diet and exercise. To determine the effectiveness of the client's efforts, the nurse should check: a) glycosylated hemoglobin level. b) fasting blood glucose level. c) serum fructosamine level. d)urine glucose level.

glycosylated hemoglobin level. Because some of the glucose in the bloodstream attaches to some of the hemoglobin and stays attached during the 120-day life span of red blood cells, glycosylated hemoglobin levels provide information about blood glucose levels during the previous 3 months. Fasting blood glucose and urine glucose levels give information only about glucose levels at the point in time when they were obtained. Serum fructosamine levels provide information about blood glucose control over the past 2 to 3 weeks.

Which combination of adverse effects should a nurse monitor for when administering I.V. insulin to a client with diabetic ketoacidosis? A. Hypocalcemia and hyperkalemia B. Hyperkalemia and hyperglycemia C. Hypokalemia and hypoglycemia D. Hypernatremia and hypercalcemia

hypokalemia and hypoglycemia Blood glucose needs to be monitored in clients receiving I.V. insulin because of the risk of hyperglycemia or hypoglycemia. Hypoglycemia might occur if too much insulin is administered. Hypokalemia, not hyperkalemia, might occur because I.V. insulin forces potassium into cells, thereby lowering the plasma level of potassium. Calcium and sodium levels aren't affected by I.V. insulin administration.

A client has been recently diagnosed with type 2 diabetes, and reports continued weight loss despite increased hunger and food consumption. This condition is called: A. anorexia. B. polyuria. C. polyphagia. D. polydipsia.

polyphagia While the needed glucose is being wasted, the body's requirement for fuel continues. The person with diabetes feels hungry and eats more (polyphagia). Despite eating more, he or she loses weight as the body uses fat and protein to substitute for glucose.

A nurse is preparing the daily care plan for a client with newly diagnosed diabetes mellitus. The priority nursing concern for this client should be: A. administering insulin routinely and as needed via a sliding scale. B. checking for the presence of ketones with each void. C. providing client education at every opportunity. D. monitoring blood glucose every 4 hours and as needed.

providing client education at every opportunity The nurse should use routine care responsibilities as teaching opportunities with the intention of preparing the client to understand and eventually manage his disease. Monitoring blood glucose, checking for the presence of ketones, and administering insulin are important when caring for a client with diabetes, but they aren't the priority of care.

A nurse is assigned to care for a postoperative client with diabetes mellitus. During the assessment interview, the client reports that he's impotent and says he's concerned about the effect on his marriage. In planning this client's care, the most appropriate intervention would be to: A. provide support for the spouse or significant other. B. suggest referral to a sex counselor or other appropriate professional. C. encourage the client to ask questions about personal sexuality. D. provide time for privacy.

suggest referral to a sex counselor or other appropriate professional The nurse should refer this client to a sex counselor or other professional. Making appropriate referrals is a valid part of planning the client's care. The nurse doesn't normally provide sex counseling.

A 32-year-old client has an appointment at the weight loss clinic where you practice nursing. She has gained 55 lbs. in the last three years and is concerned about developing Type 2 diabetes mellitus, especially since her parents both have developed the disorder. What are the conditions which contribute to developing metabolic syndrome? Choose all correct options.

• Abdominal obesity • Elevated blood glucose levels Some experts believe that diabetes in adults is one consequence of metabolic syndrome, which includes elevated blood glucose levels, hypertension, hypercholesterolemia, and abdominal obesity.

Which of the following is a characteristic of diabetic ketoacidosis (DKA)? Select all that apply.

• Elevated blood urea nitrogen (BUN) and creatinine • Rapid onset • More common in type 1 diabetes Explanation: DKA is characterized by an elevated BUN and creatinine, rapid onset, and it is more common in type 1 diabetes. Hyperglycemic hyperosmolar nonketotic syndrome (HHNS) is characterized by the absence of urine and serum ketones and a normal arterial pH level.


Ensembles d'études connexes

taxes, retirement, and other insurance concepts.

View Set

Peds- acute infectious emergencies; hepatology; congenital; diarrhea; GI; abuse

View Set

CCNA Introduction to Networks Chapter 5

View Set

Chapter 5. Analyzing the Marketing Environment

View Set

Promotion and Advertising / Chapter 22

View Set

Aggression & Violence Chapter 25

View Set